SlideShare a Scribd company logo
1 of 1091
Krok 1
For medical students
III course
• An electron micrograph shows a
cell-to-cell adhesion consisting, in
each cell, of an attachment
plaque. The intercellular space is
filled with electron-dense
substance including
transmembrane fibril structures.
Specify this adhesion:
• A *Desmosome
• B Synapse
• C Tight junction
• D Nexus
• E Adherens junction
• The action of microbial toxins
on the cells caused significant
damage of the glycocalyx.
What function of the cell
membrane will be significantly
disordered?
• A *Receptor
• B Transport
• C Respiratory
• D Formation of contact
• E Protective
• The secretory granules appear
and disappear in the cytoplasm
of pancreatic cells during the
secretory cycle. Which structural
elements these granules can be
included?
• A *To inclusions
• B To microfilaments
• C To lysosomes
• D To exocytosis vacuoles
• E To granular endoplasmic
reticulum
• Significant reduction of the
protein synthesis in hepatocytes
resulted by the long-term effects
of the toxic substances on the
body. What organelles are most
affected by intoxication?
• A *Granular endoplasmic
reticulum
• B Mitochondria
• C Microtubules
• D Lysosomes
• E Golgi apparatus
• In the tissue culture the
nucleoluses were damaged by
the radioactive irradiation.
Recovery of what organelles in
the cytoplasm becomes
problematic?
• A *Ribosomes
• B Lysosomes
• C Endoplasmic reticulum
• D Microtubule
• E Golgi apparatus
• The structure of the ribosome
was disordered in the cells. What
process is primarily affected?
• A *Protein synthesis (translation)
• B Protein synthesis (transcription)
• C Synthesis of carbohydrates
• D Synthesis of lipids
• E Synthesis of minerals
• The researchers destroyed the
structure of one of the cell parts
in a scientific experiment. As
result the cell lost the ability to
division. What structure was
broken?
• A *Centrosome
• B Glycocalix
• C Plastic complex
• D Microfibrille
• E Mitochondria
• The patient was hospitalized in the hospital with
poisoning. It was established that the
detoxification processes were disordered in the
liver. Which organelles of hepatocytes were
injured?
• A *Agranular endoplasmic reticulum
• B Mitochondria
• C Granular endoplasmic reticulum
• D Golgi apparatus
• E Ribosomes
• The organelles, which consist of cisterns that
flattened in the center and extended on the
periphery and small vesicles, were founded at the
electron microphotography of the nervous cells.
What are these organelles?
• A *Golgi apparatus
• B Centrioles
• C Lysosomes
• D Peroxisomes
• E Mitochondria
• The abnormal biopolymers were founded in the
body cells of the child (7 years) with congenital
"storage diseases". What is the kind of
organelles?
• A *Lysosomes
• B Ribosomes
• C Granular endoplasmic reticulum
• D Mitochondria
• E Peroxisomes
• A high content of hydrolytic enzymes in the
cytoplasm was founded during the examination.
Which organelle activity shows this fact?
• A *Lysosomes
• B Mitochondria
• C Polysomes
• D Endoplasmic reticulum
• E Centrioles
• The organelle that is the big polyprotease
complex and consists of tubular and two
regulatory parts that are located at both ends of
organelle was representing on electronic photo.
The function of this organelle is proteolysis. Name
this organelle.
•
• A *Proteasome
• B Centrioles
• C Inclusion
• D Ribosome
• E Golgi complex
• The vesicles with peroxide oxidation enzymes -
catalase, peroxidase (0,05-1,5 microns in
diameter) were revealed in the cytoplasm of
hepatocytes during the histochemical
investigation. What are these organelles?
• A * Peroxisomes
• B Lysosomes
• C Melanosomes
• D Liposomes
• E Phagosomes
• Low level of albumins and fibrinogen was
detected in the patient's blood. Decreased
activity of what organelle of the liver hepatocytes
can cause it?
• A *Rough endoplasmic reticulum
• B Smooth endoplasmic reticulum
• C Mitochondria
• D Golgi complex
• E Lysosomes
•
•
• There is a large quantity of carbohydrates in the
dietary intake of a human. What structures will be
seen in the cytoplasm of hepatocytes?
• *Glycogen granules
• Lipid droplets
• One large lipid drop
• Lipofuscin inclusions
• Increasing of ribosome quantity
• At what component of the cell the lysosomes
formation takes place?
• *At the Golgi apparatus
• At the nucleus
• At the ribosomes
• At the mitochondria
• At the organizing cell center
• The microtubules contain the protein:
• *Tubulin
• Desmin
• Dynein
• Calmodulin
• Vimentin
• Labeled aminoacids alanine and tryptophane
were introducted to a mouse in order to study
localization of protein biosynthesis in its cells.
Around what organellas will the accumulation of
labeled aminoacids be observed?
• A *Ribosomes
• B Agranular endoplasmic reticulum
• C Cell centre
• D Lysosomes
• E Golgi apparatus
• Ultramicroscopic examination of "dark"
hepatocytes population in the cell cytoplasm
detected a developed granular endoplasmic
reticulum. What function has this organelle in
these cells?
• A *Synthesis of blood plasma proteins
• B Carbohydrate synthesis
• C Detoxification
• D Bile production
• E Calcium ion depositing
• The hysterectomy was made to a woman 67 years
old because of a tumor. At the histological
investigation of this tumor the multipolar mitoses
were founded at the tumor cells (chromosome
disjunction to more than two cellular poles).
Disorder of what chromosoms resulted formation
of the multipolar mitosis?
• A *Centriolies
• B Secondary lysosoms
• C Smooth endoplasmic reticulum
• D Rough endoplasmic reticulum
• E Peroxisoms
• The cell was treating by a substance, which blocks
the nucleotide phosphorylation process in the
mitochondria. What process will be broken?
• A. *ATP resynthesize
• B. Synthesis of mitochondrial proteins
• C. Oxidative phosphorylation
• D. Integration of functional protein molecules
• E. Fragmentation of large mitochondria smaller
• The cell of the laboratory animal was overdosed
with Roentgen rays. As a result albuminous
fragments formed in the cytoplasm. What cell
organoid will take part at their utilization?
• A *Lysosomes
• B Endoplasmic reticulum
• C Cells centre
• D Golgi complex
• E Ribosome
•
• The histone proteins synthesis was artificially
blocked in the cell. What cell structure will be
damaged?
• A *Nuclear chromatin
• B Nucleolus
• C Golgi apparatus
• D Cell membrane
• E Nuclear envelope
• The cell without the nucleus and nucleolus is
presented at the electron microphotography. The
chromosomes are free, the centrioles migrate to
the poles. In which phase of the cell cycle is the
cell?
• A *In prophase
• B In anaphase
• C In metaphase
• D In telophase
• E In interphase
• 2012
•
• The culture of the tumor cells was affected by
colchicine, which inhibits formation of the
proteins-tubulins that are necessary for the
spindle apparatus formation. What stage of the
cell cycle will be affected?
• A *Mitosis
• B Presynthetic period
• C Synthetic period
• D Postsynthetic period
• E G0 - phase
•
• A human somatic cells in the metaphase mitosis
was founded in the histological preparation. How
many chromosomes the metaphase plate consist
of in case if each chromosome has two sister
chromatids.
• A *46 chromosomes
• B 92 chromosomes
• C 23 chromosomes
• D 48 chromosomes
• E 24 chromosomes
•
• The cells whose nuclei contain sex chromatin
(Barr corpuscle) were founded in the amniotic
fluid during the investigation that obtained by
amniocentesis (amniotic membrane puncture).
What could it mean?
• A *Development of the female sex fetus
• B Development of the male sex fetus
• C Genetic abnormalities in fetal development
• D Trisomy
• E Рolyploidy
•
• It is known that the synthesis of the proteins-
tubulins is blockade under the colchicine action.
What stage of the cell cycle will be disordered in
this case?
• *Postsynthetic period of the interphase
• Prophase of mitosis
• Synthetic period of the interphase
• Formation of the metaphase plate
• Presynthetic period of the interphase
• A very low content of heterochromatin at the
hepatocyte nuclei founded during microscopic
study. What functional state of the cell does it
mean?
• A *Increase of protein synthesis
• B Apoptosis
• C Necrosis
• D Reduction of protein synthesis
• E Entry into mitosis
•
• Compaction of the nucleus was observed on the
electron microphotography of the cells taken
from animals after a chemical exposure. What is
the state of the nucleus in the cell?
• A *Karyopyknosis
• B Mitosis
• C Amitosis
• D Polyploidy
• E Meiosis
• At the electronic microphotography the nucleus is
surrounded by the thin lamina of biological
membrane. What is it name?
• A * Nuclear envelope
• B Plasmolemma
• C Cytolemma
• D Nuclear pore
• E Pore complex
• The navy-blue chromatin granules were founded
at the nucleus on the slide that is stained by
hematoxylin and eosin. What is the stage of the
cell cycle?
• A * Interphase
• B Prophase
• C Metaphase
• D Anaphase
• E Telophase
•
•
• While studying maximally spiraled chromosomes
of human karyotype the process of cell division
was stopped in the following phase:
• A *Metaphase
• B Prophase
• C Interphase
• D Anaphase
• E Telophase
• Moving of the daughter chromatids to the poles
of the cell is observed in the mitotically dividing
cell. On what stage of the mitotic cycle is this
cell?
• A *Anaphase
• B Metaphase
• C Telophase
• D Prophase
• E Interphase
• The study of mitotic cycle phases of onion root
revealed the cell, in which the chromosomes are
situated in the equatorial plane, forming a star.
What stage of the cell mitosis is it?
• A *Metaphase
• B Anaphase
• C Telophase
• D Interphase
• E Prophase
• During the postsynthetic period of mitotic cycle
the synthesis of proteins - tubulins, which take
part in the mitosis formation, was destroyed. It
can cause the impairment of:
• A *Chromosome separation
• B Cytokinesis
• C Duration of mitosis
• D Chromosome despiralization
• E Chromosome spiralization
• There are three periods of the cell cycle
interphase. What process is take place during the
S-period?
• A *Replication of the DNA
• B Meiosis
• C Cytokinesis
• D Mitosis
• E Аmitosis
• EMBRYOLOGY. COMPARATIVE
EMBRYOLOGY OF BIRDS AND
MAMMALS
•
• The chicken embryo at the stage of the
mesoderm differentiation to somites and
splanchnotom was revealed in the histological
preparation. What material is the axial skeleton
developing from?
• A *Sclerotome
• B Dermatome
• C Nephrotome
• D Splanchnotom
• E Myotome
• The gametes precursors (gonoblasts) were
revealed in the embryo at 2nd - 3rd weeks of
embryogenesis. Where do these cells
differentiate?
• A *In the yolk sac
• B In the mesenchyme
• C In the embryonic ectoderm
• D In dermatomes
• E In the embryonic endoderm
• Study of the biopsy material of an embryo
revealed a zone of developmental abnormality in
a somite. The zone was located close to the
endoderm and the notochord. What formations
may have abnormal development in case of
pregnancy continuation?
• A *Skeletal tissues
• B Genito-urinary system
• C Skeletal striated muscle tissue
• D Cardiac striated muscle tissue
• E Fibrous connective tissue of the skin
• The sclerotome was destroyed in the bird embryo
during the experiment. Disorder of what
structure can be caused by this manipulation?
• A * Axial skeleton
• B Skin connective tissue
• C Internal organs stroma
• D Gonadal stroma
• E Chord
• The myotome was destroyed in the rabbit embryo
during the experiment. Disorder of what
structure can be caused by this manipulation?
• A * Skeletal muscle
• B Axial skeleton
• C Skin connective tissue
• D Smooth muscle
• E Serous membranes
• The outer germ layer (ectoderm) was destroyed
in the frog embryo during the experiment. What
morphological structure of this embryo will be
not developed?
• A * Epidermis
• B Somites
• C Nephrotome
• D Splanchnotom
• E Bone tissue
• The embryo on the stage of early gastrulation was
founded during the forensic medical expertise of
a woman who died in the road accident. Name
the place of the embryo localization in case of its
normal development.
• A *Uterus wall
• B Ampullar part of the oviduct
• C Uterus part of the oviduct
• D Ovary
• E Abdominal cavity
•
• The embryo composed by to blastomeres was
founded during the microscopic investigation of
female reproductive organs removed during the
operation. Name the place of the embryo
localization in case of its normal development.
• A * Oviduct, close to its ampullar part
• B Oviduct, close to its uterus part
• C Uterus cavity
• D Abdominal cavity
• E Ovary
• The process of a zygote cleavage ends with the
blastula formation. What type of blastula is
typical for a human?
• A * Blastocyst
• B Coeloblastula
• C Discoblastula
• D Amphiblastula
• E Morula
• The human embryo that is not attached to the
endometrium was found in the uterus cavity.
What is the stage of embryogenesis?
• A * Blastocyst
• B Zygote
• C Morula
• D Gastrula
• E Neurula
• The human ovum represented on histological
preparation. There are small amount of yolk
inclusions in the cytoplasm of it. Identify the type
of ovum.
• A *Secondary isolecithal
• B Isolecithal
• C Telolecithal
• D Alecithal
• E Centrolecital
•
• Chronic poisoning of 38-year-old miner by
cadmium compounds resulted in male sterility.
Semen analysis revealed the inability of sperm to
move. Damage of what cytoskeleton components
may be responsible for this pathology?
• A *Axoneme microtubules
• B Actin microfilaments
• C Intermediate filaments
• D Microtubule spindle
• E Actomyosine complex
•
• The primary Hensen's node hasn’t formed in the
embryo during the gastrulation. The development
of what axial organ will be inhibiting?
• A * Chorda
• B Neural crest
• C Neural groove
• D Neural tube
• E Mantle layer of the neural tube
•
• 2013
•
• The human blastocyst implantation begins. What
is the period of embryogenesis, which starts at
the same time with implantation?
• A * Gastrulation
• B Invagination
• C Differentiation
• D Histogenesis
• E Cleavage
•
• The chorion is determined at the microscopic
examination of the embryo membranes. What is
the main function of this organ?
• A * Exchange of substances between the mother
and fetus.
• B Hematopoietic.
• C Production of the amniotic fluid.
• D Formation of the primordial germ cells.
• E Formation of the lymphocytes
• The early gastrulation of the human embryo
occurs by delamination of the embryoblast. At
what structure does the nervous system rudiment
dispose?
• A * At the epiblast
• B At the trophoblast
• C At the hypoblast
• D At the marginal zone of the hypoblast
• E At the central zone of the hypoblast
• The hydramnios was diagnosed at the pregnant
women during the ultrasound examination.
Disorder of what extraembryonic organ function
can result in such pathological condition?
• A *Amniotic membrane
• B Chorion
• C Placenta
• D Yolk sac
• E Allantois
• The embryonic shield with two layers of cells
(ectoderm and entoderm) was revealed in the
human embryo taken from spontaneous
abortion. At what stage of embryonic
development was an embryo?
• A * Gastrulation
• B Cleavage
• C Progenesis
• D Organogenesis
• E -
• It is known that the megalocytes may appear in
the human peripheral blood. When the
appearance of these cells in the blood is normal?
• A * In the embryonic period
• B At the age of 1 year
• C At the age from 1 to 30 years
• D In old age
• E During the pregnancy
• The human embryo implantation
in the uterine wall (during the 7th
day) is the one of the critical
periods of embryogenesis. What
gastrulation process is occurs in
embryoblast during this period?
• A * Delamination
• B Migration
• C Epiboly
• D Invagination
• E Neurulation
•
• "A person was born in a shirt." What kind of
"shirt" is referred in this proverb about?
• A * Amniotic
• B Yolk
• C Serous
• D Chorionic
• E Trophoblastic
•
• 2011-2005
•
• Disorder of the cerebrum development was
founded during the ultrasonic scanning of the
pregnant woman. The chronic alcoholism is in
anamnesis. To what critical period of the
embriogenesis this pathology is conform?
• *15-20 weeks of embriogenesis
• 20-24 weeks of embriogenesis
• 7-8 days of embriogenesis
• 3-8 weeks of embriogenesis
• Neonatal period
•
• The mesenchyme at the yolk-sac wall was
destroyed on the early stage of embryogenesis.
What consequences can develop as result of this
manipulation?
• *Disorder of the blood vessels formation
• Disorder of embryo trophisity
• Disorder of metabolic products excretion
• Disorder of the allantois formation
• Disorder of the amnion formation
In course of a conditional experiment, the
development of mesenchyme cells was
completely inhibited. Development of the
following muscular tissue will be disturbed:
• A *Smooth muscle tissue
• B Neural muscle tissue
• C Epidermal muscle tissue
• D Cardiac muscle tissue
• E Skeletal muscle tissue
• TISSUES
• The villus of the small intestine is covered by the
tissue that consist only of the cells, which form a
layer on the basement membrane. The tissue
does not contain blood vessels. What tissue
covers the surface of the villus?
• A * Epithelial tissue
• B Cemented irregular fibrous connective tissue
• C Dense irregular connective tissue
• D Smooth muscle tissue
• E Reticular tissue
•
• The structures of tight junction between the
epithelial cells were affected during the
experiment. What function of the epithelium will
be disordered?
• A * Mechanical
• B Absorption
• C Vitamin D-producing
• D Secretory
• E Excretory
• Microslide contains the preparation of a gland
composed of several secretory saccule-shaped
parts that open in the common excretory duct.
What gland is it?
• A *Simple branched alveolar gland
• B Compound branched alveolar gland
• C Simple unbranched alveolar gland
• D Compound unbranched alveolar gland
• E Simple branched tubular gland
•
• The secretory parts of the apocrine glands
contain myoepithelial cells/ what is the function
of these cells?
• A *Contractive
• B Secretory
• C Protective
• D Regenerative
• E Supporting
•
• A scheme presents an exocrine gland that has
unbranched excretory duct with a terminal part
in form of a saccule opening into the duct. How is
this gland called according to the morphological
classification of exocrine glands?
• A *Simple unbranched alveolar
• B Compound branched alveolar
• C Simple branched tubular
• D Compound unbranched alveolar
• E Compound unbranched alveolar tubular
•
• The patient is complaining of the black spots
appearance on the face. Upon examination, it
was founded that the appearance of these spots
is associated with the violation of the sebaceous
glands secretion. What type of secretion is
characteristic for these glands?
• A *Holocrine
• B Merocrine
• C Macroapocrine
• D Microapocrine
• E Merocrine and microapocrine
•
• The gland that consists from two acinary
secretory parts are opening into common
excretory duct was founded in the skin. What is
the kind of a gland?
• A *Simple branched alveolar gland
• B Compound branched alveolar gland
• C Simple unbranched alveolar gland
• D Compound unbranched alveolar gland
• E Simple branched tubular gland
• The patient blood was taken for analysis. The 30%
of red blood cells with an irregular shape were
founded. Clinically, this phenomenon is described
as:
• A * Pathological poikylocytosis
• B Anisocytosis
• C Physiological poikylocytosis
• D Macrocytosis
• E Microcytosis
•
• The acute decline of the hemoglobin was
revealed in the patient's blood during the
examination at the clinic. What is the function of
blood will be disordered?
• A * Respiratory
• B Humoral
• C Homeostatic
• D Protective
• E Trophic
•
• The anemia developed at the patient 50 years old
with chronic nephritis. What was the most likely
cause of the anemia at this patient?
• A * Decreasing of erythropoietin production
• B Absence of gland
• C Lack of vitamin B12
• D Disorders of porphyrin synthesis
• E Immunological damage cells - precursors of
erythropoiesis
•
• In the red bone marrow the blood cells, which
develop, are located by the islands. Some of the
islands associated with macrophages. What blood
cells are developed in these islands?
• A * Erythrocytes
• B Precursors of T- and B-lymphocytes
• C Monocytes
• D Platelets
• E Basophilic granulocytes
•
• The reduced hemoglobin amount was revealed in
the blood test. What function of the blood will be
disordered?
• A * Transport of gases
• B Transport of hormones
• C Providing immunity
• D Clotting
• E Transport of nutrients
• In the blood of a 26-year-old man it was revealed
18% of erythrocytes of the spherical, ball-
shaped, flat and thorn-like shape. Other
erythrocytes were in the form of the concavo-
concave disks. How is such phenomenon called?
• A *Physiological poikylocytosis
• B Pathological poikylocytosis
• C Physiological anisocytosis
• D Pathological anisocytosis
• E Erytrocytosis
• The 12,5% erythrocytes with diameter greater
than 8 microns and 12.5% erythrocytes with
diameter less than 6 microns were revealed in the
patient's blood. The rest of red blood cells have a
diameter of 7,1–7,9 microns. What is the name of
this phenomenon?
• A * Physiological anisocytosis
• B Pathological anisocytosis
• C Physiological poikilocytosis
• D Pathological poikilocytosis
• E Erythrocytosis
•
• 2001-2011
•
• Transfusion of Rh-positive blood to Rh-negative
patient results in the formation of Rh antibodies
and hemolysis. What blood cells are the carrier of
Rh factor?
•
• A *Erythrocytes
• B Platelets
• C Lymphocytes
• D Monocytes
• E Neutrophils
•
• The signs of inflammation: pain, redness, edema
as signs of immediate hypersensitivity were
revealed at the child around the skin wound.
Which blood cells are causing these changes?
• A * Basophils
• B Eosinophils
• C Neutrophils
• D Lymphocytes
• E Monocytes
•
• According to results of the blood stains studying
at the crime scene the forensic pathologist
determined that it was female blood. What signs
were on the base of this conclusion?
• A * Presence of the satellite in the neutrophils
nuclei
• B Presence of the microcytes and macrocytes
• C Poikilocytosis
• D Presence of the eosinophils specific granules
• E According to the number of red blood cells
• The cells with the histamine and heparin granules
in the cytoplasm were defined in the blood
smear. What is the kind of the cell?
• A * Basophils
• B Neutrophils
• C Eosinophils
• D Monocytes
• E Erythrocytes
•
•
• The cells, which accounts for 0,5% of the total
leukocytes number with the S-shaped curved
nucleus and metachromatic colored granules in
the cytoplasm, were founded in the patient's
blood smear. What are these cells?
• A * Basophils
• B Neutrophils
• C Eosinophils
• D Monocytes
• E Lymphocytes
•
• The numerous plasma cells plasmocytes were
revealed in the blood of a 16 years old girl with
autoimmune inflammation of the thyroid gland.
With the proliferation and differentiation of what
blood cells the increase of the plasmocytes
number is associated?
• A * B-lymphocytes
• B T-helper
• C Tissue basophils
• D T-killer
• E T-suppressor
•
• The B-lymphocytes were marked with the tracer
in the experiment. The foreign protein was
injected under the skin of the animal. Which cells
in the connective tissue will contain this tracer?
• A * Plasmocytes
• B T-lymphocytes
• C Macrophages
• D Tissue basophils
• E Fibroblasts
•
• One of the blood cells populations was selectively
stimulated in the experiment. Permeability of
blood vessels was significantly increased as result
of it. That leads to edema of perivascular tissue
development end deceleration of the blood
clotting. Which blood cells were stimulated?
• A * Basophils
• B Erythrocytes
• C Platelets
• D Eosinophils
• E Lymphocytes
• The rounded cells with the segmented nuclei are
predominating from the leukocytes in the smear
of peripheral blood. The granules of their
cytoplasm stained both acidic and basic dyes.
What are these cells?
• A * Segmented neutrophils
• B Basophils
• C Eosinophils
• D Young neutrophils
• E Monocytes
•
• The large cells with low-basophilic cytoplasm and
bean-shaped nucleus were founded in a blood
smear. The cell is the largest from visible in the
visual field. What are the cells?
• A * Monocytes
• B Macrophages
• C Plasmocytes
• D Middle lymphocytes
• E Small lymphocytes
•
• 2013
•
• To determine the functional activity of blood cells
the suspension of the microorganisms was
introduced into a test tube containing leukocyte
mass. Inside of what cells the phagocytized
bacteria will be detected?
• A * Neutrophils and monocytes
• B Lymphocytes and basophils
• C Lymphocytes and eosinophils
• D Monocytes and lymphocytes
• E Lymphocytes and neutrophils
•
• The neutrophils were defined during the study of
the connective tissue microslide. What is the
function of these cells in the tissues?
• A * Phagocytosis of microorganisms
• B Trophic
• C Support function
• D Regulation of sthe mooth muscle cells
contraction
• E Dilatation of the blood vessels.
•
• It is known that plasmocytes produce specific
antibodies against of the antigen. The number of
plasmocytes is increases after the antigen
introduction. At the expense of which blood cells
the plasmocytes number will increase?
• A * B-lymphocytes
• B T-lymphocytes
• C Monocytes
• D Basophils
• E Eosinophil
•
• 2012
•
• The increasing of the total leukocytes number
was revealed in the general blood analysis of a
patient with pneumonia. What is the name of this
phenomenon?
• A * Leukocytosis
• B Anemia
• C Leukopenia
• D Anisocytosis
• E Poikilocytosis
• A live vaccine injected into a human organism.
Increased activity of what connective tissue cells
can be expected?
• A *Plasma cells and lymphocytes
• B Macrophages and fibroblasts
• C Pigmented cells and pericytes
• D Adipocytes and adventitial cells
• E Fibroblasts and labrocytes
•
• The chromatin of one of the neutrophils nucleus
segments is forming the drumstick. What is the
name of this structural formation?
• A * Barr's body
• B Lyon’s body
• C Decondensed chromatin
• D Euchromatin
• E Pacinian corpuscles
•
• The helminthiasis was diagnosed at the 6 year
child. What changes of the leukogram can be
expected.
• A * Increasing of the eosinophils number
• B Increasing of the neutrophils number
• C Reducing the eosinophils number
• D Increasing of the monocytes number
• E Increasing of the lymphocytes number
•
• The nurse complains of hands injury that
resembls the eczema. She said that after
streptomycin injection( she makes it to the
patient) the skin itching is increased and the
vesicles with the watery fluid appear on the skin.
The symptoms disappear during the holidays. The
blood test was made on suspicion of allergic
reaction. Increasing the number of what blood
cells can be detected?
• A * Eosinophilic leukocytes
• B Basophilic leukocytes
• C Monocytes
• D Neutrophilic leukocytes
• E Lymphocytes
•
• The 20% large (diameter 20 mm), rounded cells
with low-basophilic cytoplasm and bean-shaped
nucleus were founded in a blood smear. Clinically,
this phenomenon is described as:
• A * Monocytosis
• B Lymphocytosis
• C Leukopenia
• D Neutrophilia
• E Reticulocytosis
•
• The cells, which make up 0.5% of the total white
blood cells number were revealed at the patient's
blood smear. They have S-shaped nucleus and
metachromatic colored granules in the
cytoplasm. What are these cells?
• A *Basophils
• B Neutrophils
• C Eosinophils
• D Monocytes
• E Lymphocytes
•
• The helminthosis was detected at the child (10
years). What changes in the WBC can be
expected?
• A * Number of eosinophils will increase
• B Number of platelets will increase
• C Number of red blood cells will increase
• D Number of segmented neutrophils will increase
• E Number of basophils will increase
•
• In the analysis investigator made an additional
conclusion that the blood belongs to the female.
Specific features of what cells structure enable us
to conclusion?
• A * Neutrophilic leukocytes
• B Erythrocytes
• C Lymphocytes
• D Monocytes
• E Basophilic leukocytes
• The cells of next morphology: intensely basophilic
cytoplasm, eccentrically placed nucleus with
chromatin, which is located in a "spoke wheel"
and highlights the cytoplasm near it – were
founded in the lymph node histological sections
of the experimental animals after the antigen
stimulation. What is the type of cell?
• A * Plasmocytes
• B Macrophages
• C Fibroblasts
• D Adipocytes
• E Tissue basophils (mast cells)
•
• Blood sampling for bulk analysis is recommended
to be performed on an empty stomach and in the
morning. What changes in blood composition can
occur if to perform blood sampling after food
intake?
• A *Increased contents of leukocytes
• B Increased contents of erythrocytes
• C Increased plasma proteins
• D Reduced contents of plateletes
• E Reduced contents of erythrocytes
•
• The graft rejection took place after the
heterotransplantation. Which blood cells provide
the process?
• A * T-killers
• B T-helpers
• C T-suppressors
• D T0- lymphocytes
• E T-memory cells
•
• The antibodies are producing at the secondary
penetration of the antigen into the organism. By
the function of what cell does it resulted?
• A *Memory cells
• B T-killers
• C T-suppressors
• D Macrophages
• E Dendrite cells
• A large number of rounded cells with segmented
nucleus (three or more segments) and small pink-
purple granules in the cytoplasm were reveale in
a blood smear of patient with pneumonia. What
is a cell?
• A *Neutrophils
• B Monocytes
• C Eosinophilic granulocytes
• D Basophilic granulocytes
• E Lymphocytes
• General blood test of a child found the
percentage of lymphocytes and neutrophils in the
ratio of 45:45. Which age may correspond to a
physiological crossroads?
• A * 5 days and 5 years
• B 14 years
• C 1 day and 3 years
• D 1 year
• E 1 day
• Presence of inflammation in the kidney observed
at a patient. Blood test was make. It was observe
a large number of cells with following cell
morphology: nucleus with 2 - 5 segments, pink –
purple granules in the cytoplasm. What is a cell?
• A *Neutrophilic granulocytes segmented
• B Monocytes
• C Lymphocytes
• D Acidophilic granulocytes
• E Basophilic granulocytes
•
• A blood test made to a patients with allergic
rhinitis. In a blood was observe a large number of
cells with following cell morphology: nucleus with
2 - 3 segments, oxyphilic cytoplasm filled with
bright pink large granules. What is a cell?
• A *Acidophilic granulocytes
• B Lymphocytes
• C Monocytes
• D Basophilic granulocytes
• E Neutrophils
•
• Among of blood cells there are cells whose
number is 3-11% of the total number of
leukocytes. The main functions of these cells –
conversion into a macrophagic system cells;
simplest phagocytosis, aged cells phagocytosis.
What are these cells?
• A *Monocytes
• B Basophils
• C Eosinophils
• D Lymphocytes
• E Neutrophils
•
• A blood test made to a patient with pneumonia.
The blood test reveal an increase in the number
of cells that have bean-shaped and rod-shaped
nucleus, small granules, some of which are dyed
basophilic other oxyphilic. Name such changes of
WBC.
• A. *WBC shift to the left
• B. WBC shift to the right
• C. Neutrocytopenia
• D. Neuthrocytosis
• E. Lymphocytosis
• Filopodium of megakaryocytes are going through
the pores of red bone marrow sinusoidal
capillaries into the lumen of blood vessels, where
they are fragmenting into individual plates. What
blood cells are formed in this way?
• A * Platelets
• B Erythrocytes
• C Lymphocytes
• D Reticulocytes
• E Monocytes
•
• In a blood was observe a large number of cells
with following cell morphology: nucleus with 2 - 3
segments, a cytoplasm filled with large granules
that at staining showing metachromasia. What is
the cell?
• A. *Basophils
• B. Neutrophils
• C Eosinophils
• D. Monocytes
• E. Red blood cells
•
• One of the blood cells populations was selectively
stimulate. As a result of it the process of blood
clotting was significantly slowed. Which blood
cells subjected to stimulation?
• A. *Basophils
• B. Platelets
• C. Monocytes
• D. Eosinophils
• E. Neutrophils
•
• A 16% rounded cell with sizes 4.5 ... 7 mm, having
a large spherical nucleus, basophilic cytoplasm
painted in a narrow border around the nucleus
were detected in a blood smear of a patient after
suffering from flu. What state of blood do they
describe?
• A. * Lymphocytopenia
• B. Monocytosis
• C. Neutrocytosis
• D. Lymphocytosis
• E. Monocytopenia
•
•
• The significant reduction of megakaryocytes was
founded during the histological investigation of
the red bone marrow punctate of 35 years old
patient. How will change the correlation of blood
cells in this case?
• A * Number of platelets will decrease
• B Number of red blood cells will decrease
• C Number of eosinophils will decrease
• D Number of neutrophils will decrease
• E Number of B-lymphocytes will decrease
•
• It is known that megalocytes can be present in
the peripheral blood of human. When these cells
can be present in the peripheral blood in norm?
• A *In the embryogenesis
• B Till the age 1 year
• C At the age from 1 to 30
• D At the old age
• E During the pregnancy
• During postembryonic haemopoiesis in the red
bone marrow the cells of one of the cellular
differon demonstrate a gradual decrease in
cytoplasmic basophilia as well as an increase in
oxyphilia, the nucleus is being forced out. Such
morphological changes are typical for the
following haemopoiesis type:
• A *Erythropoiesis
• B Lymphopoiesis
• C Neutrophil cytopoiesis
• D Eosinophil cytopoiesis
• E Basophil cytopoiesis
•
• The megakaryocyte with demarcation channels in
the peripheral part of the cytoplasm was
determined at the electron microphotographs of
the red bone marrow. What is the function of
these structures?
• A * Formation of platelets
• B Increasing of the cell surface
• C Increase of the ion channels number
• D Cell division
• E Cell destruction
• The cells of granulocytic series were revealed at
the biopsy material of red bone marrow. Specify
what changes occur in the nucleus during the
differentiation of these cells?
• A * Segmentation
• B Polyploidisation
• C Pyknosis
• D Enucleation
• E Increasing the size
• The cells, differentiation of which is characterized
by pycnosis and nucleus removing, were revealed
in the myeloid tissue punctate of a 6 years old
child. What kind of hematopoiesisis characterized
by such morphological changes?
• A * Erythropoiesis
• B Thrombocytopoiesis
• C Lymphocytopoiesis
• D Granulocytopoiesis
• E Monocytopoiesis
• A newborn baby has the disorder of the thymus
development. What type of the hematopoiesis
will be disordered?
• A * Lymphocytopoiesis
• B Monocytopoiesis
• C Erythropoiesis
• D Granulocytopoiesis
• E Thrombocytopoiesis
• After the radioactive exposure a patient has stem
cells disorder. The regeneration of what cells of
friable connective tissue will be damaged?
• A. *Macrophages
• B. Pericytes
• C. Fibroblasts
• D. Pigment cells
• E. Adipocytes
•
• In course of an experiment, a big number of stem
cells of red bone marrow was in some way
destruct. Regeneration of which cell populations
in the loose connective tissue will be inhibit?
• A *Of macrophages
• B Of fibroblasts
• C Of pigment cells
• D Of lipocytes
• E Of pericytes
•
• Formation of structurally complex - In the
development of red blood cells in lymphoid tissue
become - erythroblastic island in the center of
which there is a cell, the processes of which
encompass the erythroblasts. Identify this cell.
• A *Macrophage
• B Fibroblasts
• C Endothelial
• D Lipocyte
• E Pericyte
•
• The least differentiated cells that are forming
populations and characterized by self-inducing,
differentiation in several ways through the
progenitor cells into functionally mature cells, are
present at every tissues during the human life (or
were present in embryogenesis). What are these
cells?
• A * Stem cells
• B Specialized cells
• C Multifunction cells
• D Cells of histogenesis
• E Blastomere
•
• The cells that characterized by
polychromatophilia, appears of meshwork
(remnants of granular endoplasmic reticulum and
free ribosomes) in the cytoplasm present in the
blood smear. Their number is normally 1-5% of
the total number of red blood cells, and
increasing their numbers is diagnostic feature of
enhanced hematopoiesis. What are these cells?
• A * Reticulocytes
• B Erythroblasts
• C Erythroblasts polychromatophilic
• D Basophilic erythroblast
• E Proerythroblast
• The method of dactylography is used extensively
in the forensic medical expertise. This method is
based on the strictly individual picture of the skin
surface made by the papillary layer of the derma.
What tissue is forming this layer of the derma?
• A * Loose connective tissue
• B Dense irregular fibrous tissue
• C Dense regular fibrous tissue
• D Reticular tissue
• E Adipose tissue
•
•
• The histamine plays a central role in the
development of allergic clinical manifestations.
What cells produce it?
• A * Mast cells
• B T lymphocytes
• C Macrophages
• D B-lymphocytes
• E Plasmocytes
•
• The local esophagus stenosis as a result of scar
formation developed at the patient after the
esophagus chemical burn. Which cells of the
loose connective tissue involved in the formation
of scars?
• A * Mature specialized fibroblasts
• B Young unspecialized fibroblasts
• C Fibrocytes
• D Myofibroblasts
• E Fibroclasts
•
• 2013
•
• A foreign body entered into the skin and leds to
inflammation development. What connective
tissue cells take place in the skin reaction agains
of the foreign body?
• A * Neutrophils, macrophages, fibroblasts
• B Macrophages
• C Melanocytes
• D Adipocytes
• E Adventicial cells
•
• Live vaccine is injected into the human body.
Increasing activity of what cells of connective
tissue can be expected?
• A *Plasmocytes and lymphocytes
• B Macrophages and fibroblasts
• C Pigmentocytes and pericytes
• D Adipocytes and adventitious cells
• E Fibroblasts and labrocytes
•
• 2012
•
• The connective tissue defects scar is developing in
the wound place during the healing. Which cells
provide the process?
• A * Fibroblasts
• B Macrophages
• C Fibrocytes
• D Mast cells
• E Melanocytes
•
• The allergic dermatitis of both hands developed
at the women as a result of the contact with the
chromium compounds at the production. Which
skin cells mainly participated in the realization of
this disease?
• A * Tissue basophils
• B Plasmocytes
• C Macrophages
• D Neutrophils
• E Lymphocytes
• A big number of the elongated cells with dense
nucleus and many lysosomes, phagosomes and
pinocytotic vacuoles in the basophilic cytoplasm
were founded in the lavage of the patient with
acute tibia wound. What are these cells?
• A * Macrophages of connective tissue
• B Fibroblasts
• C Fibrocytes
• D Plasmocytes
• E Tissue basophils
•
• In course of an experiment a big number of stem
cells of red bone marrow was in some way
destructed. Regeneration of which cell
populations in the loose connective tissue will be
inhibited?
• A *Of macrophags
• B Of fibroblasts
• C Of pigment cells
• D Of lipocytes
• E Of pericytes
•
• A significant number of red bone marrow stem
cells was destroyed at the experiment. Renewal
of what cells of connective tissue will be
inhibited?
• A * Macrophages
• B Fibroblasts
• C Pigment cells
• D Adipocytes
• E Pericytes
•
• An inflammation is characterized by the dilation
of the blood capillaries at the site of injury,
decreased blood circulation, increased vascular
permeability. Which of the following cells play the
primary role in this processes?
• A * Tissue basophils
• B Fibroblasts
• C Plasmocytes
• D Eosinophils
• E Macrophages
•
•
• 2005-2011
•
• Low concentration of the specific antibodies was
founded in the blood of a patient with the
influenza. The fuction of what cell of connective
tissue is inhibited?
• *Plasmocyte
• Lymphocyte
• Macrophage
• Macrophage
• Labrocyte
•
• The microscopic examination of wound lavage of
a patient with acute wound process of his shin
revealed big contents of irregular extended-
formed cells, with tough nucleus, the basophilic
cytoplasm of which includes many lysosomes,
phagosomes and pinocytotic bubbles. What cells
were find out in the wound?
• A. Tissue basophils
• B. *Connective tissue macrophages
• C. Fibrocytes
• D. Fibroblasts
• E. Plasmocytes
•
• A bleeding accompanied by slow blood clotting
(at normal number of platelets in blood tests)
occurred in women after the limb injury. What
substance of loose connective tissue prevents the
blood clotting process? By what cell does it
produced?
• A. * Heparin, tissue basophils
• B. Heparin, macrophages
• C. Histamine, tissue basophils
• D. Heparin, plasma cells
• E. Heparin, fibroblast
• The allergic reaction in the form of urticaria
developed at a patient (formations of the vesicles
under the epidermis as result of the plasma
release into loose connective tissue). What
substance of mast cells causes an increase of
blood vessel permeability?
• A. * Histamine
• B. Heparin
• C. Hyaluronic acid
• D. Chondroitin sulfate type A
• E. Chondroitin sulfate type C
• Leading role in the vascular
phase of inflammation the
histamine plays. Which cell of
loose connective tissue produces
histamine?
• A. * Tissue basophils
• B. Fibroblasts
• C. Plasma cell
• D. Fibrocyte
• E. Macrophage
•
• The drug ketotifen, which is able
to inhibit the histamine release
from a particular cell type, used
for long-term therapy in patients
with asthma. Name these cells.
• A *Mast cells
• B Lymphocytes
• C Eosinophils
• D Macrophages
• E Plasma cells
• In allergic rhinitis (inflammation
of the nasal mucosa) increasing
the number of basophils in the
connective tissue of the mucous
accompanied by oedema of the
tissues. With what function of
the tissue basophils does it
connected?
• A * Biogenic amines synthesis
• B Intercellular substance production
• C Phagocytosis
• D Antibody production
• E Heat production
•
•
• The lower limb was injured
during the athlete training.
Traumatologist made the
diagnosis: the tendon rupture.
What type of connective tissue
the tendon belongs to?
• A * Dense regular fibrous tissue
• B Dense irregular fibrous tissue
• C Loose connective tissue
• D Reticular tissue
• E Cartilage tissue
• The damaged Achilles tendon
function was renewed after the
treating. What is the mechanism
of a tendon regeneration?
• A * Synthesis of the collagen fibers
• B Synthesis of the hyaline cartilage
• C Formation of the adipose tissue
• D Synthesis of the fibrous cartilage
• E Replacements of the place of
injury by the muscle tissue
• Decreased blood supply to the
organs causes hypoxia that
activates fibroblasts function.
Volume of what elements is
increased in this case?
• A *Intercellular substance
• B Vessels of microcircular stream
• C Nerve elements
• D Parenchyme elements of the
organ
• E Lymphatic vessels
• The substance that disorders the
collagen fibers formation was
introduced to the animals. How
will change the tendon
properties in this case?
• A * The tendon hardness to a rupture
will decrease
• B No change
• C The tendon elasticity will decrease
• D The tendon hardness to a rupture and
elasticity will decrease
• E The tendon hardness to a rupture will
increase but elasticity will decrease
• The caesarean section was
performed to a patient. The
uterus wall was cut and the fetus
was disengaged. What is the
mechanism of uterus healing in
the area the myometrium injury?
• A * Formation of the connective
scar
• B Neoplasms of the smooth
muscular tissue
• C Formation of the striated muscle
• D Proliferation of the myosatelits
• E Hypertrophy of the smooth
muscle cells
• Decreased blood supply to the
organs causes hypoxia that
activates fibroblasts function.
Volume of what elements is
increased in this case?
• A *Intercellular substance
• B Vessels of microcircular stream
• C Nerve elements
• D Parenchymatous elements of
the organ
• E Lymphatic vessels
•
• The hyaluronidase increase
occurs at a patient under the
action of bacteria. How the
intracellular substance
permeability will change in this
case?
• A. *Increase of permeability
• B. No effect
• C. Reduce of permeability
• D. Slowing of metabolism
• E. Reduce the content of
glycosaminoglycans
• The local resorption of some bones
hard tissues was noticed by the
physician zt the roentgenogram of
the patient 57 years old. With the
increased activity of what cells can
be associated these changes?
• A * Osteoclasts
• B Chondroblasts
• C Osteocytes
• D Osteoblasts
• E Chondrocytes
• The signs of regenerative process
(callus) are present in the
histological preparation of the
bones. What tissue is forming the
described structure?
• A * Membrane reticulated bone
• B Loose connective tissue
• C Reticulartissue
• D. Epithelial tissue
• E Splenial bone
• A tissue was represented in the
histological slide. The cells of this
tissue don’t have the processec.
Each one of them conteins tens of
nuclei and one surface with
corrugated zone. Through this zone
the secretion of hydrolytic enzymes
is going. What tissue is presented in
histological slide?
• A * Bone tissue
• B Cartilage tissue
• C Epithelial tissue
• D Nervous tissue
• E Muscular tissue
• The ossification resulted by the rise
of the calcium level in the bones
was revealed at the worker of the
enterprise, which produces
vanadium compounds. With activity
of what cells can it be associated?
• A * Osteoblast
• B Osteocytes
• C Osteoclasts
• D Chondrocytes
• E Fibroblasts
•
• The resorption of bone was
revealed at the patient. With the
increased activity of what cells can
it be associated?
• A * Osteoclast
• B Osteoblasts and osteoclasts
• C Osteocytes and osteoblasts
• D Osteoblasts
• E Osteocytes
• The patients with a diagnosis of the
clavicle fracture was admitted into a
hospital. What cellular elements will
take part in the regeneration of the
bone tissue?
• A * Osteoblasts
• B Osteoclasts
• C Osteocytes
• D Chondrocytes
• E Fibroblasts
•
• The excessive loss of bone tissue
mass (it reflects the development of
osteoporosis) is observed at the
elderly people. Activation of what
bone cells can cause the
development of this disease?
• A * Osteoclast
• B Osteoblasts
• C Macrophages
• D Tissue basophils
• E Osteocytes
• The bones knitting at the fracture
place is resulted by the osteoblasts
activation. What is the function of
these cells?
A. A*Formation of the osteoid substance
and transformation into osteocytes
B. Formation of the connective tissue
primary, from which will be bone
formation
C. Formation of the cartilage isogenic group
D. Proliferation and calcification
E. Formation of the bone and
transformation into osteoclasts
• The hydroxyapatite crystals
deposition along the collagen fibers
takes place during the intercellular
substance of bone calcification.
Presence of alkaline phosphatase in
the intercellular substance in
necessary for this process. What cell
produces this enzyme?
• A *Osteoblasts
• B Osteocytes
• C Osteoclasts
• D Chondroblast
• E Chondrocytes
• The disorder of the motive
functions resulted by the age
changes of the hyaline cartilage was
revealed at the patient 70 years old.
What age changes caused limitation
of the joints movement?
• A * Deposition of calcium in the
intercellular substance
• B Increasing the number of isogenic
groups
• C Increasing the number of cartilage
cells
• D Thickening of the perichondrium
• E Increasing the hydrophilicity of the
ground substance
• The isogenic cell groups were
revealed at the histological slide of
the cartilage. What cells are the
primary in the formation of these
groups?
• A * Chondrocytes I type.
• B Chondroblasts
• C Prechondroblasts
• D Chondrocytes II type
• E Chondrocytes III type
•
• The tissue was represented in a
histological slide. The cells of this
tissue are situated singly or by
isogroups and the fibrous structures
of the intercellular substance are
not visible. What tissue is
represented in the slide?
• A * Hyaline cartilage
• B Smooth muscle tissue
• C Epithelial tissue
• D Fibrous cartilage
• E Bone tissue
•
• The tissue is visible at the larynx
tumor biopsy material. The cells of
this tissue are situated singly or
forming the isogenic cells group
located in the same plane. The
presence of elastic and collagen
fibers is detected histologically.
Which structures could develop this
tumor?
• A * Elastic cartilage
• B Hyaline cartilage
• C Fibrous cartilage
• D Smooth muscle tissue
• E Bone tissue
• The two slides were proposed to
the student. At the 1st one there is
the elastic cartilage (stained by the
orcein), at the 2nd one – the hyaline
cartilage (stained by the
hematoxylin-eosin). According to
what features can we tell one from
the other?
• A * Presence of the elastic fibers
• B Presence of the cells isogenic
groups
• C Presence of young cartilage zone
• D Presence of the perichondrium
• E Presence of amorphous substance
• The elongation of the bones is
stopped after the puberty. Decrease
of what cell proliferation results
stopping of the bones elongation?
A. *Chondroblasts
B. Osteocytes
C. Osteoclasts
D. Fibroblasts
E. Neutrophils
• During the study of histological
preparations of the airways it was
founded that the supporting structures
of the large and medium bronchi wall
is different according to their tissue
composition. What tissue replaced the
hyaline cartilage in the fibro-
cartilaginous shell of secondary
bronchi (compared to the large
bronchi)?
• A * Elastic cartilage
• B Loose connective
• C Hyaline cartilage
• D Smooth muscle
• E Fibrous cartilage
• Synovial fluid normally doesn’t
contains collagen. Analysis of
synovial fluid of patients suffering
from rheumatism, showed the
presence of various types of
collagen. Specify the type of erosive
tissue if the synovial fluid is
determined by collagen type II.
•
• A *Articular surface cartilage
• B Synovial membrane
• C Vascular endothelium
• D Myocytes vessels
• E Bone epiphysis
•
• The articular cartilage, as it is
known, doesn’t have the
perichondrium. What kind of
growth is possible at this cartilage
during the regeneration process?
• A * Interstitial
• B Appositional
• C By application
• D Appositional and interstitial
• E It is not growing
•
• In course of indirect histogenesis of
tubular bone tissue a plate is
formed between epiphyseal and
diaphyseal ossification centres that
provides further lengthwise growth
of bones. What structure is it?
• A *Metaphyseal plate
• B Osseous cuff
• C Osseous plate
• D Osteon
• E Layer of interior general plates
•
• The disorder of cartilage tissue
regeneration resulted by the injury
of the undifferentiated cartilage
cells was observed at the patient
with severe upper extremity injury.
What cells were injured?
• A * Cells of the perichondrium inner
layer
• B Cells of the perichondrium outer
layer
• C Isogenic groups cells
• D Cells of the young cartilage zone
• E Cells originating from blood
vessels
• The cartilage part of the rib was
damaged as result of trauma. At the
cost of what layer of perichondrium
the regeneration of the cartilage
will take place?
• A * Chondrogenic
• B Fibrous
• C Elastic
• D Collagen
• E Connective tissue
• The destruction of the thin
myofilaments is observed after the
action of the hydrolytic enzymes.
What structures were damaged?
• A * Actin myofilaments
• B Myosin myofilaments
• C Tonofibrils
• D Tropocollagen complexes
• E Nucleoprotein complexes
•
• The tissue was represented at the
histological slide. The structural unit
of this tissue is the muscle fiber,
which consist of the myosymplast
and satellitocytes and is covered by
a basement membrane. What tissue
is characterized by this structure?
• A * Skeletal striated muscle tissue
• B Smooth muscle tissue
• C Cardiac muscle tissue
• D Loose connective tissue
• E Reticular tissue
•
• Negative environmental factors
have caused the dysfunction of
myosatellite cells. What function of
the whole muscle fiber is likely to be
changed in this case?
• A *Regeneration
• B Contraction
• C Trophism
• D Contractile thermogenesis
• E Relaxation
•
• The cesarean section was
performed to a patient. The uterus
wall was cut a significant extent and
fetus released. By what mechanism
the myometrium regeneration will
be in the area of wound?
• A *Formation of connective tissue
scar
• B Formation of new smooth muscle
• C Formation of striated muscle
fibers
• D Proliferation of myosatellitocytes
• E Hypertrophy of smooth muscle
cells
• A microspecimen of the
submandibular salivary gland shows
some basket-shaped cells
concentrated around the acinus and
excretory ducts. These cells
surround bases of the serous cells
and are called myoepitheliocytes.
These cells relate to the following
tissue:
• A *Muscular tissue
• B Epithelial tissue
• C Neural tissue
• D Special connective tissue
• E Loose fibrous connective tissue
• In course of a conditional
experiment the development of
mesenchyma cells was completely
inhibited. Development of the
following muscular tissue will be
disturbed:
• A *Smooth muscular tissue
• B Neural muscular tissue
• C Epidermal muscular tissue
• D Cardiac muscular tissue
• E Skeletal muscular tissue
•
• A big number of the intermediate
microfilaments that contain desmin
were revealed at the cytoplasm of
the cell, which is spindle-shaped
and has a rod-like nucleus. What
tissue these cells belong to?
• A * Muscle
• B Nervous
• C Epithelial
• D Connective
• E –
• The destruction of the thick
myofilaments is observed after
mechanical injury of striated muscle
fibers. Where is the localization of
the pathological changes?
• A * In the disk A
• B In the disk I
• C In the disk A half
• D In the A and I disks
• E In the disk I half
•
• Patient with injured muscles of the
lower extremities was admitted to
the traumatology department. Due
to what cells is reparative
regeneration of the muscle fibers
and restoration of the muscle
function possible?
• A *Satellite-cells
• B Myoblasts
• C Myofibroblasts
• D Fibroblasts
• E Myoepithelial cells
• It is known, that calcium ions, along
with other factors, provide
contraction of the muscle tissue.
With what structures calcium
interact during contraction?
• A. *Protein troponin of thin fibrils
• B. Protein myosin of thick fibrils
• C. Protein actin of thin fibrils
• D. Actomyosine complex of
sarcolemma
• E. Protein sequestrin
• The skeletal muscle fibres are
damaged due to injury. What
structures can be the source of
reparative regeneration of skeletal
muscle fibres?
• A. *Myosatellitocytes
• B. Myofibrils
• C. Miofilaments
• D. Sarcolemma
• E. Endomysium
• Name the protein that is the main
component of the thin
myofilaments of the muscle cells.
•
A. *Actin
B. Tubulin
C. Dunein
D. Desmin
E. Keratin
•
• General increase of the skeletal
muscle mass is takes place at the
sportsman. Hypertrophy of what
kind of the muscle fibers at the
skeletal muscle will be dominating
in this case?
A. *White muscle fiber
B. Red muscle fiber
C. Intermediate muscle fiber
D. Cardiac muscle fiber
E. Smooth myocytes
• A sensitive neural ganglion consists
of roundish neurocytes with one
extension that divides into axon and
dendrite at some distance from the
perikaryon. What are these cells
called?
•
• A *Pseudounipolar
• B Unipolar
• C Bipolar
• D Multipolar
• E Apolar
•
• The toxic substances action violates
the mechanism of the nerve
impulses transmission at the
experiment. What structure
provides the implementation of this
function?
• A * Synapse
• B Neurolemma
• C Neurofibril
• D Mitochondria
• E Nissl’s substance
•
• The degeneration of nerve fibers
accompanied by breakage of axial
cylinders and myelin destruction
may develop in case of traumatic
injury of the upper limb. What
neural structures will make the
myelin renewal during the
regeneration?
• A * Neurolemmocytes (Schwann
cells)
• B Mesaxon
• C Perineurium
• D Endoneurium
• E Astrocytes
• The Nissl body lysis is observed in
case of neuron functional
exhaustion. It is resulted by decay of
basophilic substance and
brightening of the neuroplasma.
With the changes of what organelle
does this phenomenon is
associated?
• A. * Granular endoplasmic
reticulum
• B. Neurofibrils
• C. Lysosomes
• D. Golgi apparatus
• E. Mitochondria
• The emergency medical assistance was
given to the patient, who had an accident.
A tourniquet had been imposed forearm to
stop the bleeding for a long time. This
resulted in a loss of sensation and
movement in the arm. Examination of the
patient allowed to establish a violation of
nerve conduction in a mixed nerve caused
by its destruction without displacement of
nerve fibers fragments. Proliferation of
what cells will provide the axial cylinders
regeneration in a mixed nerves?
• A. * Schwann's cell
• B. Fibrous astrocytes
• C. Neurons
• D. Ependymal cell
• E. Protoplasmatic astrocytes
•
• To elderly patients turned to
neurologist with complaints about the
difficulty of the neck movements,
numbness and loss of right hand
sensation. Examination of the patient
revealed a deformation of the spine in
the lower cervical and upper thoracic
part of the spinal cord caused by
excessive deposition of calcium salts.
What cells dysfunction resulted the
loss of sensitivity in the hand?
• A. * Pseudounipolar neurons
• B. The neurons of the spinal cord
anterior horns
• C The neurons of the spinal cord
posterior horns
• D. The neurons of the spinal cord
lateral horns
• E. Pyramidal neurons
•
• The multipolar neurocytes are
observed in the gray matter on the
histological specimen of the
cerebellum cross-section. According
to what morphological features are
these cells attributed to multipolar?
• A * Amount of the processes
• B Length of the processes
• C Shape of the terminal axon
extension
• D Shape of the perikaryon
• E Size of the cells
•
• The cells of neuroglia are forming
from two sources: ectoderm and
mesenchyme. What cells have the
mesenchymal origin?
• A * Microgliocytes
• B Ependymal cell
• C Fibrillar astrocytes
• D Protoplasmatic astrocytes
• E Oligodendrocyte
• The spongioblasts were destroyed
during the experiment. What
violations of nerve tissue will occur
during of it differentiation?
• A *Disorder of neuroglia development
• B Disorder of white matter of the
spinal cord development
• C Disorder of gray matter of the spinal
cord development
• D Disorder of the spinal ganglion
development
• E Disorder of the cerebrum
development
• The channel that is surrounded by a
dense layer of cells we can see on
the cross-section of the spinal cord.
What are these cells and where
they are located in the body?
• A * Ependymal cells, all cerebrum
ventricles
• B Astrocytes, all cerebrum ventricles
• C Oligodendrocytes, gray matter of the
spinal cord
• D Astrocytes, white matter of the
spinal cord
• E Neuron, myelin sheath
•
• The area of peripheral nerve longer
than 10 cm was damaged after a
mechanical injury. The motor
activity of the upper extremity is
impaired because of it. An
allotransplantation of the nerve was
proposed to the patient. What glial
cells take will take a part in the
regeneration of the damaged area.
• A. * Neurolemmocytes
• B. Fibrous astrocytes
• C. Protoplasmatic astrocytes
• D. Ependymal cells
• E. Microgliocytes
• The convulsions that associated
with incomplete nerve fibers
myelination can easily arise at the
children of the first year. What glial
cells most associated with this
situation?
• A. * Oligodendrocyte
• B. Ependymal cells
• C. Fibrous astrocytes
• D. Microglia cells
• E. Protoplasmatic astrocytes
• The toxic substances action violates
the mechanism of nerve impulses
transmission in the experiment.
What structure is provide the
implementation of this function?
• A * Synapse
• B Neurolemma
• C Neurofibrills
• D Mitochondria
• E Chromophilic substance
• The concentrically oriented dark
and bright rings, which are located
around the axial cylinder, were
revealed in the electron microscope
pictures of the myelin fiber cross-
section. What substances are
forming the dark rings?
• A *Lipids
• B Protein
• C Vitamins
• D Carbohydrates
• E Water
•
• The several axial cylinders with
mesaxon were revealed in the
electron microscope pictures of the
fiber cross-section. What is the kind
of fiber?
• A * Nonmyelinated nerve fiber
• B Reticular fiber
• C Collagen fiber
• D Elastic fiber
• E Myelinated nerve fiber
•
• The fiber with one of its shell that is
black and has a light notches
located at an angle to the axial
cylinder was founded in the nervous
tissue slide colored by osmic acid.
What is a shell?
• A *Myelin
• B Neurolemma
• C Aksolemma
• D Basement membrane
• E Endonevrium
• Special histology
• As a result of a trauma a patient has
damaged anterior roots of spinal
cord. What structures have been
affected?
• A *Axons of motoneurons and axons
of neurons of lateral horns
• B Central processes of sensitive
neurons of spinal ganglions
• C Peripheral processes of sensitive
spinal ganglions
• D Axons of neurons of lateral horns
• E Dendrites of neurons of spinal
ganglions
• One of sections of central nervous
system has layer wise arrangement
of neurocytes. Among them there
are cells of the following forms:
stellate, fusiform, horizontal,
pyramidal. What section of central
nervous system is this structure
typical for?
• A Cortex of cerebrum
• B Spinal cord
• C Cerebellum
• D Medulla oblongata
• E Hypothalamus
• As a result of an injury, the integrity
of the anterior spinal cord root was
broken. Specify the neurons and
their processes that had been
damaged:
• A *Axons of motor neurons
• B Motor neuron dendrites
• C Axons of sensory neurons
• D Dendrites of sensory neurons
• E Dendrites of association neurons
• Alcohol intoxication, as a rule, is
accompanied by the coordination of
movements’ disorder and
imbalance caused by the damage of
cerebellum structural elements. The
function of what cells of cerebellum
is affected first of all?
A. *Purkinje’s cells
B. Basket cells
C. Betz cells
D. Stellate cells
E. Granule cells
• Cerebellar cortex is revealed in a
specimen impregnated with silver
salts. It includes pyriform, basket,
stellate and granule cells. What
neurons constitute molecular layer?
A. *Basket, small and large stellate
cells
B. Stellate and pyramidal cells
C. Granule cells and large stellate
cells
D. Large stellate cells and spindle
cells
E. Pyriform cells
• Cerebellar cortex is revealed in a
specimen impregnated with silver
salts. It includes pyriform, basket,
stellate and granule cells. Which
from mentioned above cells is
efferent neuron of cerebellum?
A. *Pyriform cells
B. Stellate cells
C. Granule cells
D. Pyramidal cells
E. Spindle cells
• In a histological specimen an organ of
nervous system is presented, which
consists of grey and white substances.
Grey substance is located on the
periphery and consists of 6 layers:
molecular, external granular, external
pyramidal, internal granular, internal
pyramidal and the layer of
polymorphic cells.
A. *Cerebral cortex
B. Cerebellum
C. Pons cerebelli
D. Spinal ganglion
E. Spinal cord
• In a histological specimen an organ
of nervous system is presented,
which consists of grey and white
substances. Grey substance is
located on the center and consists
of efferent neurons, projection
neurons and interneurons. Name
this organ.
A. *Spinal cord
B. Pons cerebelli
C. Cerebral cortex
D. Cerebellum
E. Spinal ganglion
• Parenchyma of the organ consists
of the nervous tissue in which
pseudounipolar neurons are
revealed. Perikaryon of the neurons
is covered with glial and connective
tissue membranes and located in
bunches. Name this organ.
A. *Spinal ganglia (sensory ganglia)
B. Vegetative ganglion
C. Epiphysis
D. Spinal cord
E. Cerebellum
• In a histological specimen an organ of
nervous system is presented, which
consists of grey and white substances.
Grey substance is located on the
center and forms butterfly. Neurons in
the grey matter locate in bunches and
form nucleuses. Which nucleus
belongs to the central part of the
vegetative nervous system?
A. *Intermediate lateral nucleus
B. Nucleus proprius of the dorsal
horn
C. Nucleus proprius of the ventral
horn
D. Nucleus thoracicus
E. Intermediate medial nucleus
•
• Precentral gyrus section of the
cerebral cortex is presented in the
histological specimen. Indicate
which layers mostly developed in
this case.
A. *Pyramidal external and internal
and layer of polymorphic cells
B. Molecular
C. External and internal granular
D. Molecular and layer of
polymorphic cells
E. Molecular, pyramidal external and
internal
• A part of the central nervous system
has layer by layer allocation of
neurocytes, among which there are
cells of such forms: stellate,
fusiform, horizontal, pyramidal.
Which part of the CNS has this
structure?
A. *Cortex of large hemispheres
B. Cerebellum
C. Hypothalamus
D. Medulla oblongata
E. Spinal cord
• 55-year- old patient has
movements’ coordination and
balance disorder as a result of
permanent using of alcohol and
developing intoxication. What
nervous structures of the CNS have
been disordered?
A. *Purkinje cells of the cerebellum
B. Basket cells of the cerebellum
C. Stellate cells of the cerebellum
D. Motor neurons of the spinal cord
E. Olive of the medulla oblongata
• 15-year-old patient enrolled in the
clinic with diagnosis poliomyelitis.
This disease is accompanied with
disorder of movements. What
nervous structures destruction can
explain this disorder?
A. *Motor neurons of the spinal cord
B. Sensory neurons of the spinal
ganglions
C. Vegetative nucleuses of the spinal
cord
D. Substantia gelatinosa
E. Neurons of the cerebellum
• A patient with poliomyelitis (which
characterized by spinal cord
damage) has disorder of skeletal
muscles function. What neurons
destruction can explain this
disorder?
A. *Motor neurons
B. Pseudounipolar
C. Associative (interneurons)
D. Pseudounipolar and associative
(interneurons)
E. Interneurons and motor
•
• In a histological specimen an organ
of nervous system is presented,
which consists of grey and white
substances. Grey substance is
located on the periphery. Neurons
in the grey matter form three layers
molecular, ganglionary (Purkinje),
and granular. What organ has this
structure?
A. *Cerebellum
B. Spinal cord
C. Cerebral cortex
D. Medulla oblongata
E. Pons
•
• A specimen, dyed by the method of
silver impregnation, is being
investigated. Pyramidal cells of
different size are seen in this
specimen. Short processes come off
their tips and lateral surfaces; one
long process comes off the base of
the cells. Name the specimen.
A. *Cerebral cortex
B. Spiral organ of the inner ear
C. Retina of the eye
D. Cortex of the cerebellum
E. Spinal ganglion
• A specimen, dyed by the method of
silver impregnation, is being
investigated. Pyriform cells with 2-3
climbing up prominent dendrites
are seen in this specimen. Name the
specimen.
A. *Cortex of the cerebellum
B. Spiral organ of the inner ear
C. Retina of the eye
D. Cerebral cortex
E. Spinal ganglion
•
• During microscopic investigation of
the CNS grey substance was
revealed.. Neurons in it form three
layers molecular, ganglionary
(Purkinje), and granular. Name
neurons which form second layer?
A. *Pyriform cells
B. Small stellate cells
C. Granule cells
D. Large stellate
E. Basket cells
•
• In the microspecimen of the spinal
cord nucleus neurons of which form
motor ending in the skeletal
muscles have to be analyzed.
Indicate this nucleus
A. *Nucleus proprius of the ventral
horn
B. Nucleus thoracicus
C. Intermediolateral nucleus
D. Nucleus proprius of the dorsal
horn
E. Nucleus proprius of grey
substance
• As a result of trauma anterior roots
of the spinal cord were damaged in
47-year-old patient. Which neurons
processes were damaged?
A. *Axons of motor somatic and
vegetative nucleuses neurons
B. Axons of sensory pseudounipolar
neurons
C. Dendrites of sensory pseudounipolar
neurons
D. Dendrites of motor and axons of
lateral horns nucleuses
E. Dendrites and axons of sensory
pseudounipolar neurons
• A histological specimen presents a
receptor zone of a sensoepithelial sense
organ. Cells of this zone are placed upon
the basal membrane and include the
following types: external and internal
receptor cells, external and internal
phalangeal cell, stem cells, external
limiting cells and external supporting cell.
The described receptor zone belongs to
the following sense organ:
•
• A *Acoustic organ
• B Visual organ
• C Gustatory organ
• D Equilibrium organ
• E Olfactory organ
•
• Vitamin A deficit results in the
impairment of twilight vision. Name
the cells that have the above-
mentioned photoreceptor function:
• A *Rod receptor cell
• B Horizontal neurocytes
• C Cone receptor cells
• D Bipolar neurons
• E Ganglion neurocytes
•
• An infectious disease caused
contractive activity of muscles that
contract and dilate eye pupil
(paralytic state). What functional
eye system was damaged?
• A *Accomodative
• B Dioptric
• C Ancillary
• D Photosensory
• E Lacrimal apparatus
•
• A 14-year-old patient has twilight
vision impairment. What vitamin
deficit takes place in the organism?
A. *A
B. B1
C. В6
D. C
E. В12
• During examination an oculist has
detected that patient has disorder
of green color perception. Which
retina cells absence may cause this
sight disorder?
• *Cone neurosensory cells
• Rod neurosensory cells
• Epithelial pigmented cells
• Bipolar neuron
• Ganglionic nerve cell
• The increased intraocular tension is
observed in the patient with
glaucoma. Secretion of aqueous
humor by the ciliary body is normal.
Injury of what structure of the
eyeball wall caused the disorder of
flow-out from the anterior
chamber?
• A *Venous sinus
• B Ciliary body
• C Choroid
• D Ciliary muscle
• E Back epithelium of cornea
• A histological specimen of the
eyeball shows a biconvex structure
connected to the ciliary body by the
fibers of the Zinn's zonule and
covered with a transparent capsule.
Name this structure:
• A *Crystalline lens
• B Vitreous body
• C Ciliary body
• D Cornea
• E Sclera
•
• In a histological specimen a
structure of eyeball detected in
which blood vessels are absent.
What structure is characterized by
this morphological sign?
A. *Cornea
B. Ciliary body
C. Choroid
D. Iris
E. Retina
•
• In the electron microphotograph of
the sense organ revealed cells
peripheral part of which constitutes
from two segments. The outer
segment has membrane half discs
and the inner one has ellipsoid. In
what organ does this structure
locate?
A. *In the organ of vision
B. In the organ of taste
C. In the organ of smell
D. In the vestibular organ
E. In the auditory organ
• In an electronic micrograph of a
sense organ hair cells are seen, on
their apical part there are short
microvilli – stereocilii and a polar
located kinocilium. For what
sensory organ are such cells typical?
A. *Vestibular organ
B. Vision organ
C. Olfactory organ
D. Hearing organ
E. Taste organ
• Damages of vascular membrane are
detected in the histological
specimen of a fetus eyeball. What
embryonic material was probably
damaged during the development
of the eye?
A. *Mesenchyme
B. Ectoderm
C. Endoderm
D. Outer layer of the eyeball
E. Internal layer of the eyeball
• In the electron microphotograph
revealed cell of neural origin.
Terminal part of the cell dendrite
has cylindrical shape and consists
from 1000 enclosed membrane
discs. What cell is this?
• *Rod cell
• Ventral horn of the spinal cord
• Sensory ganglia neuron
• Cerebral cortex neuron
• Cone cell
•
• As a result of trauma 30-year-old
man has damaged mucosal
membrane which covers upper part
of upper conchae. To what
consequences did this lead?
• *Disorder of smell substances
perception
• Disorder of air moistening
• Disorder of secretory activity of
goblet cells
• Disorder of air warming
• Disorder of air moistening and
warming
• As a result of head trauma 32-year-
old patient has damaged cristae
ampullaris. Which stimuli
perception have been disordered?
• *Angular movement
• Vibration
• Gravitation
• Linear movement
• Vibration and gravitation
•
• A lot of people with age have
clouding of the lens
(phacoscotasmus or cataract) It
became opaque that leading to the
partial blindness. What optical and
chemical properties of the lens
fibers protein will be disordered?
• *Crystalline
• Vitrein
• Dinein
• Rhodopsin
• Iodopsin
• A patient has taken high doses of
streptomycin and consequently
became deaf. The function of what
cells of the inner ear was damaged
in this case?
A. *Hair
B. Phalangeal
C. Pillar
D. Deiters’
E. Connective tissue
•
• A boxer has disturbance in smell
after a trauma of the nose. The
damage of what cells may cause the
loss of smell?
A. *Neurosensory
B. Supporting epithelial cells
C. Basal epithelial cells
D. Microvillous epithelial cells
E. Brush cells
• On the electron microphotograph
neural origin cell is presented. This
cell constitutes part of mucosa
membrane epithelium. Distal part
of peripheral process of the cell has
knob-like expansion from which
arise 10-15 cilia. What cell is it?
A. *Olfactory cell
B. Bipolar neuron of spinal ganglion
C. Sensor epithelial cell of organ of
taste
D. Rode cell
E. Cone cell
• An infectious disease caused
contractive activity of muscles that
contract and dilate eye pupil
(paralytic state). What functional
eye system was damaged?
• A *Accommodative
• B Dioptric
• C Ancillary
• D Photosensory
• E Lacrimal apparatus
• Underdeveloped epithelium of
cornea is observed in the
histological specimen of a fetus’
eyeball. A part of what embryonal
layer was probably affected in the
process of embryogenesis?
A. *Ectoderm
B. Endoderm
C. Mesoderm
D. Outer layer of the eyeball
E. Inner layer of the eyeball
• Ciliary body was damaged in the
patient. Function of what eye
apparatus will suffer in this case?
A. *Accommodative
B. Protective
C. Trophic
D. Photosensitive
E. Dioptric
• Patient with eye trauma appealed
to the doctor. During examination of
corneal epithelium was revealed
changes. What epithelium was
damaged?
A. *Stratified squamous non
keratinized
B. Simple pseudostratified
C. Stratified squamous keratinized
D. Stratified cuboidal
E. Stratified cylindrical
• In the histological specimen of the
eyeball wall, structure which
consists of three neurons chain is
revealed. Bodies of these neurons
form outer, inner nuclear and
ganglion cell layers. Which eye
component has this morphologic
structure?
A. *Retina
B. Iris
C. Sclera
D. Choroid
E. Ciliary body
• Transplantation of the cornea was
done to the patient. Which
peculiarities of the cornea structure
give expectation to engraftment of
it?
A. *Absence of blood and typical
lymphatic vessels
B. Presence of stratified anterior
epithelium
C. A huge innervations
D. Presence of connective tissue
E. Presence of simple squamous
epithelium
• Patient has appealed to the
ophthalmologist with complaints of
hurt in his eyes, which revealed itself
after long presence of the patient in
the field during dust storm. Doctor
diagnosed superficial injuries of the
external corneal epithelium. Which
cells provide regeneration of the
injured epithelium?
A. *Basal cells
B. Cells of stratum corneum
C. Cells of stratum granulosum
D. Cells of stratum spinosum
E. Cells of stratum lucidum
• Study of fingerprints
(dactylography) is used by
criminalists for personal
identification as well as for
diagnostics of genetic
abnormalities, particularly Dawn's
disease. What layer of skin
determines individuality of
fingerprints?
• A *Dermopapillary
• B Horny
• C Reticular
• D Clear (stratum lucidum
epidermis)
• E Basal
•
• A patient complains of dryness of
head skin, itching, fragility and loss
of hair. After examination he was
diagnosed with seborrhea.
Disturbed activity of which cells
caused this condition?
• A *Cells of sebaceous glands
• B Cells of sudoriferous glands
• C Epithelial cells
• D Adipocytes
• E Melanocytes
•
• In an electronic micrograph of skin
epidermis among the cells of
cuboidal form dendritic cells are
detected. In their cytoplasm Golgi
apparatus is well-developed; there
are a lot of ribosomes and
melanosomes. Name these cells.
A. *Melanocytes
B. Keratinocytes
C. Cells of Langerhans
D. Merkel’s cells
E. Mast cells
• Trauma of skin reticular layer was
happened. At the expense of what
cells differon activity regeneration
of this layer will happen?
A. *Fibroblastic
B. Macrophagic
C. Lymphoblastic
D. Neuroblastic
E. -
• Under the radiation influence
epidermal cells of the stratum
basale were damaged. What
function of the epidermis will be
weakening or depressed first of all?
A. *Regenerative
B. Protective
C. Absorptive
D. Barrier
E. Dielectric
• With age human skin undergoes
changes, which may declare
themselves by reduction of skin
elasticity. What structures of
connective tissue provide skin
elasticity most of all?
A. *Collagen and elastic fibers
B. Ground substance
C. Epidermis cells
D. Connective tissue cells
E. Reticular fiber
• In the course of experiment on a
frog embryo the external embryonic
layer – ectoderm – has been
destroyed. Which of the following
morphological structures has not
been developed henceforth?
•
A. *Epidermis
B. Somites
C. Nephrotome
D. Splanchnotome
E. Sclerotome
• One of the surgery rules is
performing sections along the so-
called Langers’ lines (lines of skin
tension). Which from mentioned
below tissues form papillary layer
(the strongest layer in the derma)?
A. *Dense irregular connective tissue
B. Reticular connective tissue
C. Loose connective tissue
D. Epithelial tissue
E. Dense regular connective tissue
• A child has abraded skin of the palm
when falling down. What
epithelium was damaged?
A. *Stratified keratinized
B. Stratified non-keratinized
C. Simple low columnar
D. Transitional
E. Simple squamous
• A patient complains of dryness of
head skin, itching, fragility and loss
of hair. After examination he was
diagnosed with seborrhea.
Disturbed activity of which cells
caused this condition?
• A *Cells of sebaceous glands
• B Cells of sudoriferous glands
• C Epithelial cells
• D Adipocytes
• E Melanocytes
• Cells with processes and dark
brown granules in the cytoplasm
were revealed in the skin epidermis
of the biopsy material. What cell are
these?
A. *Melanocytes
B. Intraepidermal macrophages
C. Keratinocytes
D. Merkel cells
E. Lymphocytes
• Some layers are absent on a limited
area of epidermis after a trauma.
Only germinative layer is preserved.
Name the cells, which will become
the main source of its regeneration.
A. *Layer of basal cells
B. Layer of spinosum cells
C. Layer of granulosum cells
D. Layer of spinous and granular cells
of undisturbed area
E. Cells of lucidum layer of
undisturbed area
• A 12-year-old patient has white
spots without a pigment on skin.
Spots have appeared after the age
of 10, constantly increase in size.
The absence of what cells of skin led
to such formations appearance?
A. *Melanocytes
B. Adipocytes
C. Fibrocytes
D. Plasma cells
E. Mast cells
• During the third week of
embryogenesis the central part of
epiblast cells (ectoderm) sags and
neurulation process begins. In
which direction will the remaining
ectodermal cells differentiate?
A. *Skin
B. Somites
C. Gut
D. Chord
E. Yolk sac
• There is histological specimen of the
skin epidermis bioptic sample taken
from the healthy adult. Cells
division can be seen in the stratum
basale. What process does provide
these cells?
A. *Physiologic regeneration
B. Differentiation
C. Adaptation
D. Reparation
E. Apoptosis
• In forensics method of
dactyloscopy is largely used. This
method is based on papillary layer
of derma which determines strictly
individual print on the skin surface.
Which tissue forms this layer of the
derma?
• *Loose irregular connective tissue
• Reticular connective tissue
• Dense irregular connective tissue
• Adipose tissue
• Dense regular connective tissue
• Stratified structure organ which
covered with stratified keratinized
squamous epithelium is represented in
the histological specimen. There is
loose connective tissue which forms
convexity in the form of papilla located
under epithelial basal membrane.
Dense irregular connective tissue
located under it and form reticular
layer. What organ has these
morphological peculiarities?
A. *Skin
B. Tongue
C. Esophagus
D. Tonsil
E. Cervix of the uterus
• There are next layers in the skin
histological specimen: stratum
basale, spinosum, granulosum,
lucidum and corneum. Which part
of the human body does this
epithelium belong?
A. *Palm skin
B. Face skin
C. Hairy part of the head skin
D. Shoulder skin
E. Thigh skin
• Skin malignant tumor was revealed
of 30-year-old patient. What
epidermis cells take place in the
immune response?
A. *T-lymphocytes
B. Keratinocytes
C. Keratinocytes and Merkel cells
D. Merkel cells
E. Stratum spinosum cells
•
• Process of dorsal mesoderm
segmentation and somites
formation was disordered in the
embryo. In what part of the skin
disorders of development are
possible?
A. *Derma
B. Hair
C. Sebaceous glands
D. Epidermis
E. Sweat glands
• Trauma of derma reticular layer
was happened. At the expense of
what cell differon activity
regeneration of this layer will
happen?
krok presentation medical. dasaaaaaassspptx
krok presentation medical. dasaaaaaassspptx
krok presentation medical. dasaaaaaassspptx
krok presentation medical. dasaaaaaassspptx
krok presentation medical. dasaaaaaassspptx
krok presentation medical. dasaaaaaassspptx
krok presentation medical. dasaaaaaassspptx
krok presentation medical. dasaaaaaassspptx
krok presentation medical. dasaaaaaassspptx
krok presentation medical. dasaaaaaassspptx
krok presentation medical. dasaaaaaassspptx
krok presentation medical. dasaaaaaassspptx
krok presentation medical. dasaaaaaassspptx
krok presentation medical. dasaaaaaassspptx
krok presentation medical. dasaaaaaassspptx
krok presentation medical. dasaaaaaassspptx
krok presentation medical. dasaaaaaassspptx
krok presentation medical. dasaaaaaassspptx
krok presentation medical. dasaaaaaassspptx
krok presentation medical. dasaaaaaassspptx
krok presentation medical. dasaaaaaassspptx
krok presentation medical. dasaaaaaassspptx
krok presentation medical. dasaaaaaassspptx
krok presentation medical. dasaaaaaassspptx
krok presentation medical. dasaaaaaassspptx
krok presentation medical. dasaaaaaassspptx
krok presentation medical. dasaaaaaassspptx
krok presentation medical. dasaaaaaassspptx
krok presentation medical. dasaaaaaassspptx
krok presentation medical. dasaaaaaassspptx
krok presentation medical. dasaaaaaassspptx
krok presentation medical. dasaaaaaassspptx
krok presentation medical. dasaaaaaassspptx
krok presentation medical. dasaaaaaassspptx
krok presentation medical. dasaaaaaassspptx
krok presentation medical. dasaaaaaassspptx
krok presentation medical. dasaaaaaassspptx
krok presentation medical. dasaaaaaassspptx
krok presentation medical. dasaaaaaassspptx
krok presentation medical. dasaaaaaassspptx
krok presentation medical. dasaaaaaassspptx
krok presentation medical. dasaaaaaassspptx
krok presentation medical. dasaaaaaassspptx
krok presentation medical. dasaaaaaassspptx
krok presentation medical. dasaaaaaassspptx
krok presentation medical. dasaaaaaassspptx
krok presentation medical. dasaaaaaassspptx
krok presentation medical. dasaaaaaassspptx
krok presentation medical. dasaaaaaassspptx
krok presentation medical. dasaaaaaassspptx
krok presentation medical. dasaaaaaassspptx
krok presentation medical. dasaaaaaassspptx
krok presentation medical. dasaaaaaassspptx
krok presentation medical. dasaaaaaassspptx
krok presentation medical. dasaaaaaassspptx
krok presentation medical. dasaaaaaassspptx
krok presentation medical. dasaaaaaassspptx
krok presentation medical. dasaaaaaassspptx
krok presentation medical. dasaaaaaassspptx
krok presentation medical. dasaaaaaassspptx
krok presentation medical. dasaaaaaassspptx
krok presentation medical. dasaaaaaassspptx
krok presentation medical. dasaaaaaassspptx
krok presentation medical. dasaaaaaassspptx
krok presentation medical. dasaaaaaassspptx
krok presentation medical. dasaaaaaassspptx
krok presentation medical. dasaaaaaassspptx
krok presentation medical. dasaaaaaassspptx
krok presentation medical. dasaaaaaassspptx
krok presentation medical. dasaaaaaassspptx
krok presentation medical. dasaaaaaassspptx
krok presentation medical. dasaaaaaassspptx
krok presentation medical. dasaaaaaassspptx
krok presentation medical. dasaaaaaassspptx
krok presentation medical. dasaaaaaassspptx
krok presentation medical. dasaaaaaassspptx
krok presentation medical. dasaaaaaassspptx
krok presentation medical. dasaaaaaassspptx
krok presentation medical. dasaaaaaassspptx
krok presentation medical. dasaaaaaassspptx
krok presentation medical. dasaaaaaassspptx
krok presentation medical. dasaaaaaassspptx
krok presentation medical. dasaaaaaassspptx
krok presentation medical. dasaaaaaassspptx
krok presentation medical. dasaaaaaassspptx
krok presentation medical. dasaaaaaassspptx
krok presentation medical. dasaaaaaassspptx
krok presentation medical. dasaaaaaassspptx
krok presentation medical. dasaaaaaassspptx
krok presentation medical. dasaaaaaassspptx
krok presentation medical. dasaaaaaassspptx
krok presentation medical. dasaaaaaassspptx
krok presentation medical. dasaaaaaassspptx
krok presentation medical. dasaaaaaassspptx
krok presentation medical. dasaaaaaassspptx
krok presentation medical. dasaaaaaassspptx
krok presentation medical. dasaaaaaassspptx
krok presentation medical. dasaaaaaassspptx
krok presentation medical. dasaaaaaassspptx
krok presentation medical. dasaaaaaassspptx
krok presentation medical. dasaaaaaassspptx
krok presentation medical. dasaaaaaassspptx
krok presentation medical. dasaaaaaassspptx
krok presentation medical. dasaaaaaassspptx
krok presentation medical. dasaaaaaassspptx
krok presentation medical. dasaaaaaassspptx
krok presentation medical. dasaaaaaassspptx
krok presentation medical. dasaaaaaassspptx
krok presentation medical. dasaaaaaassspptx
krok presentation medical. dasaaaaaassspptx
krok presentation medical. dasaaaaaassspptx
krok presentation medical. dasaaaaaassspptx
krok presentation medical. dasaaaaaassspptx
krok presentation medical. dasaaaaaassspptx
krok presentation medical. dasaaaaaassspptx
krok presentation medical. dasaaaaaassspptx
krok presentation medical. dasaaaaaassspptx
krok presentation medical. dasaaaaaassspptx
krok presentation medical. dasaaaaaassspptx
krok presentation medical. dasaaaaaassspptx
krok presentation medical. dasaaaaaassspptx
krok presentation medical. dasaaaaaassspptx
krok presentation medical. dasaaaaaassspptx
krok presentation medical. dasaaaaaassspptx
krok presentation medical. dasaaaaaassspptx
krok presentation medical. dasaaaaaassspptx
krok presentation medical. dasaaaaaassspptx
krok presentation medical. dasaaaaaassspptx
krok presentation medical. dasaaaaaassspptx
krok presentation medical. dasaaaaaassspptx
krok presentation medical. dasaaaaaassspptx
krok presentation medical. dasaaaaaassspptx
krok presentation medical. dasaaaaaassspptx
krok presentation medical. dasaaaaaassspptx
krok presentation medical. dasaaaaaassspptx
krok presentation medical. dasaaaaaassspptx
krok presentation medical. dasaaaaaassspptx
krok presentation medical. dasaaaaaassspptx
krok presentation medical. dasaaaaaassspptx
krok presentation medical. dasaaaaaassspptx
krok presentation medical. dasaaaaaassspptx
krok presentation medical. dasaaaaaassspptx
krok presentation medical. dasaaaaaassspptx
krok presentation medical. dasaaaaaassspptx
krok presentation medical. dasaaaaaassspptx
krok presentation medical. dasaaaaaassspptx
krok presentation medical. dasaaaaaassspptx
krok presentation medical. dasaaaaaassspptx
krok presentation medical. dasaaaaaassspptx
krok presentation medical. dasaaaaaassspptx
krok presentation medical. dasaaaaaassspptx
krok presentation medical. dasaaaaaassspptx
krok presentation medical. dasaaaaaassspptx
krok presentation medical. dasaaaaaassspptx
krok presentation medical. dasaaaaaassspptx
krok presentation medical. dasaaaaaassspptx
krok presentation medical. dasaaaaaassspptx
krok presentation medical. dasaaaaaassspptx
krok presentation medical. dasaaaaaassspptx
krok presentation medical. dasaaaaaassspptx
krok presentation medical. dasaaaaaassspptx
krok presentation medical. dasaaaaaassspptx
krok presentation medical. dasaaaaaassspptx
krok presentation medical. dasaaaaaassspptx
krok presentation medical. dasaaaaaassspptx
krok presentation medical. dasaaaaaassspptx
krok presentation medical. dasaaaaaassspptx
krok presentation medical. dasaaaaaassspptx
krok presentation medical. dasaaaaaassspptx
krok presentation medical. dasaaaaaassspptx
krok presentation medical. dasaaaaaassspptx
krok presentation medical. dasaaaaaassspptx
krok presentation medical. dasaaaaaassspptx
krok presentation medical. dasaaaaaassspptx
krok presentation medical. dasaaaaaassspptx
krok presentation medical. dasaaaaaassspptx
krok presentation medical. dasaaaaaassspptx
krok presentation medical. dasaaaaaassspptx
krok presentation medical. dasaaaaaassspptx
krok presentation medical. dasaaaaaassspptx
krok presentation medical. dasaaaaaassspptx
krok presentation medical. dasaaaaaassspptx
krok presentation medical. dasaaaaaassspptx
krok presentation medical. dasaaaaaassspptx
krok presentation medical. dasaaaaaassspptx
krok presentation medical. dasaaaaaassspptx
krok presentation medical. dasaaaaaassspptx
krok presentation medical. dasaaaaaassspptx
krok presentation medical. dasaaaaaassspptx
krok presentation medical. dasaaaaaassspptx
krok presentation medical. dasaaaaaassspptx
krok presentation medical. dasaaaaaassspptx
krok presentation medical. dasaaaaaassspptx
krok presentation medical. dasaaaaaassspptx
krok presentation medical. dasaaaaaassspptx
krok presentation medical. dasaaaaaassspptx
krok presentation medical. dasaaaaaassspptx
krok presentation medical. dasaaaaaassspptx
krok presentation medical. dasaaaaaassspptx
krok presentation medical. dasaaaaaassspptx
krok presentation medical. dasaaaaaassspptx
krok presentation medical. dasaaaaaassspptx
krok presentation medical. dasaaaaaassspptx
krok presentation medical. dasaaaaaassspptx
krok presentation medical. dasaaaaaassspptx
krok presentation medical. dasaaaaaassspptx
krok presentation medical. dasaaaaaassspptx
krok presentation medical. dasaaaaaassspptx
krok presentation medical. dasaaaaaassspptx
krok presentation medical. dasaaaaaassspptx
krok presentation medical. dasaaaaaassspptx
krok presentation medical. dasaaaaaassspptx
krok presentation medical. dasaaaaaassspptx
krok presentation medical. dasaaaaaassspptx
krok presentation medical. dasaaaaaassspptx
krok presentation medical. dasaaaaaassspptx
krok presentation medical. dasaaaaaassspptx
krok presentation medical. dasaaaaaassspptx
krok presentation medical. dasaaaaaassspptx
krok presentation medical. dasaaaaaassspptx
krok presentation medical. dasaaaaaassspptx
krok presentation medical. dasaaaaaassspptx
krok presentation medical. dasaaaaaassspptx
krok presentation medical. dasaaaaaassspptx
krok presentation medical. dasaaaaaassspptx
krok presentation medical. dasaaaaaassspptx
krok presentation medical. dasaaaaaassspptx
krok presentation medical. dasaaaaaassspptx
krok presentation medical. dasaaaaaassspptx
krok presentation medical. dasaaaaaassspptx
krok presentation medical. dasaaaaaassspptx
krok presentation medical. dasaaaaaassspptx
krok presentation medical. dasaaaaaassspptx
krok presentation medical. dasaaaaaassspptx
krok presentation medical. dasaaaaaassspptx
krok presentation medical. dasaaaaaassspptx
krok presentation medical. dasaaaaaassspptx
krok presentation medical. dasaaaaaassspptx
krok presentation medical. dasaaaaaassspptx
krok presentation medical. dasaaaaaassspptx
krok presentation medical. dasaaaaaassspptx
krok presentation medical. dasaaaaaassspptx
krok presentation medical. dasaaaaaassspptx
krok presentation medical. dasaaaaaassspptx
krok presentation medical. dasaaaaaassspptx
krok presentation medical. dasaaaaaassspptx
krok presentation medical. dasaaaaaassspptx
krok presentation medical. dasaaaaaassspptx
krok presentation medical. dasaaaaaassspptx
krok presentation medical. dasaaaaaassspptx
krok presentation medical. dasaaaaaassspptx
krok presentation medical. dasaaaaaassspptx
krok presentation medical. dasaaaaaassspptx
krok presentation medical. dasaaaaaassspptx
krok presentation medical. dasaaaaaassspptx
krok presentation medical. dasaaaaaassspptx
krok presentation medical. dasaaaaaassspptx
krok presentation medical. dasaaaaaassspptx
krok presentation medical. dasaaaaaassspptx
krok presentation medical. dasaaaaaassspptx
krok presentation medical. dasaaaaaassspptx
krok presentation medical. dasaaaaaassspptx
krok presentation medical. dasaaaaaassspptx
krok presentation medical. dasaaaaaassspptx
krok presentation medical. dasaaaaaassspptx
krok presentation medical. dasaaaaaassspptx
krok presentation medical. dasaaaaaassspptx
krok presentation medical. dasaaaaaassspptx
krok presentation medical. dasaaaaaassspptx
krok presentation medical. dasaaaaaassspptx
krok presentation medical. dasaaaaaassspptx
krok presentation medical. dasaaaaaassspptx
krok presentation medical. dasaaaaaassspptx
krok presentation medical. dasaaaaaassspptx
krok presentation medical. dasaaaaaassspptx
krok presentation medical. dasaaaaaassspptx
krok presentation medical. dasaaaaaassspptx
krok presentation medical. dasaaaaaassspptx
krok presentation medical. dasaaaaaassspptx
krok presentation medical. dasaaaaaassspptx
krok presentation medical. dasaaaaaassspptx
krok presentation medical. dasaaaaaassspptx
krok presentation medical. dasaaaaaassspptx
krok presentation medical. dasaaaaaassspptx
krok presentation medical. dasaaaaaassspptx
krok presentation medical. dasaaaaaassspptx
krok presentation medical. dasaaaaaassspptx
krok presentation medical. dasaaaaaassspptx
krok presentation medical. dasaaaaaassspptx
krok presentation medical. dasaaaaaassspptx
krok presentation medical. dasaaaaaassspptx
krok presentation medical. dasaaaaaassspptx
krok presentation medical. dasaaaaaassspptx
krok presentation medical. dasaaaaaassspptx
krok presentation medical. dasaaaaaassspptx
krok presentation medical. dasaaaaaassspptx
krok presentation medical. dasaaaaaassspptx
krok presentation medical. dasaaaaaassspptx
krok presentation medical. dasaaaaaassspptx
krok presentation medical. dasaaaaaassspptx
krok presentation medical. dasaaaaaassspptx
krok presentation medical. dasaaaaaassspptx
krok presentation medical. dasaaaaaassspptx
krok presentation medical. dasaaaaaassspptx
krok presentation medical. dasaaaaaassspptx
krok presentation medical. dasaaaaaassspptx
krok presentation medical. dasaaaaaassspptx
krok presentation medical. dasaaaaaassspptx
krok presentation medical. dasaaaaaassspptx
krok presentation medical. dasaaaaaassspptx
krok presentation medical. dasaaaaaassspptx
krok presentation medical. dasaaaaaassspptx
krok presentation medical. dasaaaaaassspptx
krok presentation medical. dasaaaaaassspptx
krok presentation medical. dasaaaaaassspptx
krok presentation medical. dasaaaaaassspptx
krok presentation medical. dasaaaaaassspptx
krok presentation medical. dasaaaaaassspptx
krok presentation medical. dasaaaaaassspptx
krok presentation medical. dasaaaaaassspptx
krok presentation medical. dasaaaaaassspptx
krok presentation medical. dasaaaaaassspptx
krok presentation medical. dasaaaaaassspptx
krok presentation medical. dasaaaaaassspptx
krok presentation medical. dasaaaaaassspptx
krok presentation medical. dasaaaaaassspptx
krok presentation medical. dasaaaaaassspptx
krok presentation medical. dasaaaaaassspptx
krok presentation medical. dasaaaaaassspptx
krok presentation medical. dasaaaaaassspptx
krok presentation medical. dasaaaaaassspptx
krok presentation medical. dasaaaaaassspptx
krok presentation medical. dasaaaaaassspptx
krok presentation medical. dasaaaaaassspptx
krok presentation medical. dasaaaaaassspptx
krok presentation medical. dasaaaaaassspptx
krok presentation medical. dasaaaaaassspptx
krok presentation medical. dasaaaaaassspptx
krok presentation medical. dasaaaaaassspptx
krok presentation medical. dasaaaaaassspptx
krok presentation medical. dasaaaaaassspptx
krok presentation medical. dasaaaaaassspptx
krok presentation medical. dasaaaaaassspptx
krok presentation medical. dasaaaaaassspptx
krok presentation medical. dasaaaaaassspptx
krok presentation medical. dasaaaaaassspptx
krok presentation medical. dasaaaaaassspptx
krok presentation medical. dasaaaaaassspptx
krok presentation medical. dasaaaaaassspptx
krok presentation medical. dasaaaaaassspptx
krok presentation medical. dasaaaaaassspptx
krok presentation medical. dasaaaaaassspptx
krok presentation medical. dasaaaaaassspptx
krok presentation medical. dasaaaaaassspptx
krok presentation medical. dasaaaaaassspptx
krok presentation medical. dasaaaaaassspptx
krok presentation medical. dasaaaaaassspptx
krok presentation medical. dasaaaaaassspptx
krok presentation medical. dasaaaaaassspptx
krok presentation medical. dasaaaaaassspptx
krok presentation medical. dasaaaaaassspptx
krok presentation medical. dasaaaaaassspptx
krok presentation medical. dasaaaaaassspptx
krok presentation medical. dasaaaaaassspptx
krok presentation medical. dasaaaaaassspptx
krok presentation medical. dasaaaaaassspptx
krok presentation medical. dasaaaaaassspptx
krok presentation medical. dasaaaaaassspptx
krok presentation medical. dasaaaaaassspptx
krok presentation medical. dasaaaaaassspptx
krok presentation medical. dasaaaaaassspptx
krok presentation medical. dasaaaaaassspptx
krok presentation medical. dasaaaaaassspptx
krok presentation medical. dasaaaaaassspptx
krok presentation medical. dasaaaaaassspptx
krok presentation medical. dasaaaaaassspptx
krok presentation medical. dasaaaaaassspptx
krok presentation medical. dasaaaaaassspptx
krok presentation medical. dasaaaaaassspptx
krok presentation medical. dasaaaaaassspptx
krok presentation medical. dasaaaaaassspptx
krok presentation medical. dasaaaaaassspptx
krok presentation medical. dasaaaaaassspptx
krok presentation medical. dasaaaaaassspptx
krok presentation medical. dasaaaaaassspptx
krok presentation medical. dasaaaaaassspptx
krok presentation medical. dasaaaaaassspptx
krok presentation medical. dasaaaaaassspptx
krok presentation medical. dasaaaaaassspptx
krok presentation medical. dasaaaaaassspptx
krok presentation medical. dasaaaaaassspptx
krok presentation medical. dasaaaaaassspptx
krok presentation medical. dasaaaaaassspptx
krok presentation medical. dasaaaaaassspptx
krok presentation medical. dasaaaaaassspptx
krok presentation medical. dasaaaaaassspptx
krok presentation medical. dasaaaaaassspptx
krok presentation medical. dasaaaaaassspptx
krok presentation medical. dasaaaaaassspptx
krok presentation medical. dasaaaaaassspptx
krok presentation medical. dasaaaaaassspptx
krok presentation medical. dasaaaaaassspptx
krok presentation medical. dasaaaaaassspptx
krok presentation medical. dasaaaaaassspptx
krok presentation medical. dasaaaaaassspptx
krok presentation medical. dasaaaaaassspptx
krok presentation medical. dasaaaaaassspptx
krok presentation medical. dasaaaaaassspptx
krok presentation medical. dasaaaaaassspptx
krok presentation medical. dasaaaaaassspptx
krok presentation medical. dasaaaaaassspptx
krok presentation medical. dasaaaaaassspptx
krok presentation medical. dasaaaaaassspptx
krok presentation medical. dasaaaaaassspptx
krok presentation medical. dasaaaaaassspptx
krok presentation medical. dasaaaaaassspptx
krok presentation medical. dasaaaaaassspptx
krok presentation medical. dasaaaaaassspptx
krok presentation medical. dasaaaaaassspptx
krok presentation medical. dasaaaaaassspptx
krok presentation medical. dasaaaaaassspptx
krok presentation medical. dasaaaaaassspptx
krok presentation medical. dasaaaaaassspptx
krok presentation medical. dasaaaaaassspptx
krok presentation medical. dasaaaaaassspptx
krok presentation medical. dasaaaaaassspptx
krok presentation medical. dasaaaaaassspptx
krok presentation medical. dasaaaaaassspptx
krok presentation medical. dasaaaaaassspptx
krok presentation medical. dasaaaaaassspptx
krok presentation medical. dasaaaaaassspptx
krok presentation medical. dasaaaaaassspptx
krok presentation medical. dasaaaaaassspptx
krok presentation medical. dasaaaaaassspptx
krok presentation medical. dasaaaaaassspptx
krok presentation medical. dasaaaaaassspptx
krok presentation medical. dasaaaaaassspptx
krok presentation medical. dasaaaaaassspptx
krok presentation medical. dasaaaaaassspptx
krok presentation medical. dasaaaaaassspptx
krok presentation medical. dasaaaaaassspptx
krok presentation medical. dasaaaaaassspptx
krok presentation medical. dasaaaaaassspptx
krok presentation medical. dasaaaaaassspptx
krok presentation medical. dasaaaaaassspptx
krok presentation medical. dasaaaaaassspptx
krok presentation medical. dasaaaaaassspptx
krok presentation medical. dasaaaaaassspptx
krok presentation medical. dasaaaaaassspptx
krok presentation medical. dasaaaaaassspptx
krok presentation medical. dasaaaaaassspptx
krok presentation medical. dasaaaaaassspptx
krok presentation medical. dasaaaaaassspptx
krok presentation medical. dasaaaaaassspptx
krok presentation medical. dasaaaaaassspptx
krok presentation medical. dasaaaaaassspptx
krok presentation medical. dasaaaaaassspptx
krok presentation medical. dasaaaaaassspptx
krok presentation medical. dasaaaaaassspptx
krok presentation medical. dasaaaaaassspptx
krok presentation medical. dasaaaaaassspptx
krok presentation medical. dasaaaaaassspptx
krok presentation medical. dasaaaaaassspptx
krok presentation medical. dasaaaaaassspptx
krok presentation medical. dasaaaaaassspptx
krok presentation medical. dasaaaaaassspptx
krok presentation medical. dasaaaaaassspptx
krok presentation medical. dasaaaaaassspptx
krok presentation medical. dasaaaaaassspptx
krok presentation medical. dasaaaaaassspptx
krok presentation medical. dasaaaaaassspptx
krok presentation medical. dasaaaaaassspptx
krok presentation medical. dasaaaaaassspptx
krok presentation medical. dasaaaaaassspptx
krok presentation medical. dasaaaaaassspptx
krok presentation medical. dasaaaaaassspptx
krok presentation medical. dasaaaaaassspptx
krok presentation medical. dasaaaaaassspptx
krok presentation medical. dasaaaaaassspptx
krok presentation medical. dasaaaaaassspptx
krok presentation medical. dasaaaaaassspptx
krok presentation medical. dasaaaaaassspptx
krok presentation medical. dasaaaaaassspptx
krok presentation medical. dasaaaaaassspptx
krok presentation medical. dasaaaaaassspptx
krok presentation medical. dasaaaaaassspptx
krok presentation medical. dasaaaaaassspptx
krok presentation medical. dasaaaaaassspptx
krok presentation medical. dasaaaaaassspptx
krok presentation medical. dasaaaaaassspptx
krok presentation medical. dasaaaaaassspptx
krok presentation medical. dasaaaaaassspptx
krok presentation medical. dasaaaaaassspptx
krok presentation medical. dasaaaaaassspptx
krok presentation medical. dasaaaaaassspptx
krok presentation medical. dasaaaaaassspptx
krok presentation medical. dasaaaaaassspptx
krok presentation medical. dasaaaaaassspptx
krok presentation medical. dasaaaaaassspptx
krok presentation medical. dasaaaaaassspptx
krok presentation medical. dasaaaaaassspptx
krok presentation medical. dasaaaaaassspptx
krok presentation medical. dasaaaaaassspptx
krok presentation medical. dasaaaaaassspptx
krok presentation medical. dasaaaaaassspptx
krok presentation medical. dasaaaaaassspptx
krok presentation medical. dasaaaaaassspptx
krok presentation medical. dasaaaaaassspptx
krok presentation medical. dasaaaaaassspptx
krok presentation medical. dasaaaaaassspptx
krok presentation medical. dasaaaaaassspptx
krok presentation medical. dasaaaaaassspptx
krok presentation medical. dasaaaaaassspptx
krok presentation medical. dasaaaaaassspptx
krok presentation medical. dasaaaaaassspptx
krok presentation medical. dasaaaaaassspptx
krok presentation medical. dasaaaaaassspptx
krok presentation medical. dasaaaaaassspptx
krok presentation medical. dasaaaaaassspptx
krok presentation medical. dasaaaaaassspptx
krok presentation medical. dasaaaaaassspptx
krok presentation medical. dasaaaaaassspptx
krok presentation medical. dasaaaaaassspptx
krok presentation medical. dasaaaaaassspptx
krok presentation medical. dasaaaaaassspptx
krok presentation medical. dasaaaaaassspptx
krok presentation medical. dasaaaaaassspptx
krok presentation medical. dasaaaaaassspptx
krok presentation medical. dasaaaaaassspptx
krok presentation medical. dasaaaaaassspptx
krok presentation medical. dasaaaaaassspptx
krok presentation medical. dasaaaaaassspptx
krok presentation medical. dasaaaaaassspptx
krok presentation medical. dasaaaaaassspptx
krok presentation medical. dasaaaaaassspptx
krok presentation medical. dasaaaaaassspptx
krok presentation medical. dasaaaaaassspptx
krok presentation medical. dasaaaaaassspptx
krok presentation medical. dasaaaaaassspptx
krok presentation medical. dasaaaaaassspptx
krok presentation medical. dasaaaaaassspptx
krok presentation medical. dasaaaaaassspptx
krok presentation medical. dasaaaaaassspptx
krok presentation medical. dasaaaaaassspptx
krok presentation medical. dasaaaaaassspptx
krok presentation medical. dasaaaaaassspptx
krok presentation medical. dasaaaaaassspptx
krok presentation medical. dasaaaaaassspptx
krok presentation medical. dasaaaaaassspptx
krok presentation medical. dasaaaaaassspptx
krok presentation medical. dasaaaaaassspptx
krok presentation medical. dasaaaaaassspptx
krok presentation medical. dasaaaaaassspptx
krok presentation medical. dasaaaaaassspptx
krok presentation medical. dasaaaaaassspptx
krok presentation medical. dasaaaaaassspptx
krok presentation medical. dasaaaaaassspptx

More Related Content

Similar to krok presentation medical. dasaaaaaassspptx

Cell Structure and Function
Cell Structure and Function Cell Structure and Function
Cell Structure and Function Fasama H. Kollie
 
CELL BIOLOGY LECTURE. Cells described in the purest of forms.
CELL BIOLOGY LECTURE. Cells described in the purest of forms.CELL BIOLOGY LECTURE. Cells described in the purest of forms.
CELL BIOLOGY LECTURE. Cells described in the purest of forms.ndugbu34
 
Prokaryotic and Eukaryotic Algal cell structure
Prokaryotic and Eukaryotic Algal cell structureProkaryotic and Eukaryotic Algal cell structure
Prokaryotic and Eukaryotic Algal cell structuregkumarimahesh
 
Cell structure and Function
Cell structure and FunctionCell structure and Function
Cell structure and Functionanjuparswan
 
Prokaryotic cell
Prokaryotic cellProkaryotic cell
Prokaryotic cellmeetkang1
 
Ingles celula animal
Ingles celula animalIngles celula animal
Ingles celula animalcasper trout
 
CELL-the-unit-of-life-8.pptx
CELL-the-unit-of-life-8.pptxCELL-the-unit-of-life-8.pptx
CELL-the-unit-of-life-8.pptxMUSharavanaPriyan
 
Structure and functions of cell organelles.pptx
Structure and functions of cell organelles.pptxStructure and functions of cell organelles.pptx
Structure and functions of cell organelles.pptxjaya1992
 
Lecture 2aThe cell 1.pdf
Lecture 2aThe cell 1.pdfLecture 2aThe cell 1.pdf
Lecture 2aThe cell 1.pdfNatungaRonald1
 
Cell Membrane & ion transport.pptx
Cell Membrane & ion transport.pptxCell Membrane & ion transport.pptx
Cell Membrane & ion transport.pptxMira426412
 
Cell signaling, regulating mechanism and structure
Cell signaling, regulating mechanism and structureCell signaling, regulating mechanism and structure
Cell signaling, regulating mechanism and structureGunJee Gj
 
Ch. 21 cells and tissues
Ch. 21 cells and tissuesCh. 21 cells and tissues
Ch. 21 cells and tissuesrupalmahesh
 
Quiz+ unit+of+life
Quiz+ unit+of+lifeQuiz+ unit+of+life
Quiz+ unit+of+lifePinakin Gore
 
Anatomy of prokaryotic cells and eukaryotic cells with differences
Anatomy of prokaryotic cells and eukaryotic cells with differencesAnatomy of prokaryotic cells and eukaryotic cells with differences
Anatomy of prokaryotic cells and eukaryotic cells with differencesHassanLatif15
 
L1 Introduction to cells.pptx
L1 Introduction to cells.pptxL1 Introduction to cells.pptx
L1 Introduction to cells.pptxAbdulkarim803288
 

Similar to krok presentation medical. dasaaaaaassspptx (20)

Prokaryotic & eukaryotic cells 3
Prokaryotic & eukaryotic cells  3Prokaryotic & eukaryotic cells  3
Prokaryotic & eukaryotic cells 3
 
Cell
CellCell
Cell
 
Cell Structure and Function
Cell Structure and Function Cell Structure and Function
Cell Structure and Function
 
CELL BIOLOGY LECTURE. Cells described in the purest of forms.
CELL BIOLOGY LECTURE. Cells described in the purest of forms.CELL BIOLOGY LECTURE. Cells described in the purest of forms.
CELL BIOLOGY LECTURE. Cells described in the purest of forms.
 
Prokaryotic and Eukaryotic Algal cell structure
Prokaryotic and Eukaryotic Algal cell structureProkaryotic and Eukaryotic Algal cell structure
Prokaryotic and Eukaryotic Algal cell structure
 
Cell structure and Function
Cell structure and FunctionCell structure and Function
Cell structure and Function
 
Prokaryotic cell
Prokaryotic cellProkaryotic cell
Prokaryotic cell
 
Ingles celula animal
Ingles celula animalIngles celula animal
Ingles celula animal
 
CELL-the-unit-of-life-8.pptx
CELL-the-unit-of-life-8.pptxCELL-the-unit-of-life-8.pptx
CELL-the-unit-of-life-8.pptx
 
Structure and functions of cell organelles.pptx
Structure and functions of cell organelles.pptxStructure and functions of cell organelles.pptx
Structure and functions of cell organelles.pptx
 
Lecture 2aThe cell 1.pdf
Lecture 2aThe cell 1.pdfLecture 2aThe cell 1.pdf
Lecture 2aThe cell 1.pdf
 
Cell Membrane & ion transport.pptx
Cell Membrane & ion transport.pptxCell Membrane & ion transport.pptx
Cell Membrane & ion transport.pptx
 
Cell signaling, regulating mechanism and structure
Cell signaling, regulating mechanism and structureCell signaling, regulating mechanism and structure
Cell signaling, regulating mechanism and structure
 
Ch. 21 cells and tissues
Ch. 21 cells and tissuesCh. 21 cells and tissues
Ch. 21 cells and tissues
 
plasmamembrane.pptx
plasmamembrane.pptxplasmamembrane.pptx
plasmamembrane.pptx
 
Cell and tissue
Cell and tissueCell and tissue
Cell and tissue
 
Quiz+ unit+of+life
Quiz+ unit+of+lifeQuiz+ unit+of+life
Quiz+ unit+of+life
 
Anatomy of prokaryotic cells and eukaryotic cells with differences
Anatomy of prokaryotic cells and eukaryotic cells with differencesAnatomy of prokaryotic cells and eukaryotic cells with differences
Anatomy of prokaryotic cells and eukaryotic cells with differences
 
Cell
CellCell
Cell
 
L1 Introduction to cells.pptx
L1 Introduction to cells.pptxL1 Introduction to cells.pptx
L1 Introduction to cells.pptx
 

Recently uploaded

Pune Airport ( Call Girls ) Pune 6297143586 Hot Model With Sexy Bhabi Ready...
Pune Airport ( Call Girls ) Pune  6297143586  Hot Model With Sexy Bhabi Ready...Pune Airport ( Call Girls ) Pune  6297143586  Hot Model With Sexy Bhabi Ready...
Pune Airport ( Call Girls ) Pune 6297143586 Hot Model With Sexy Bhabi Ready...tanu pandey
 
On Starlink, presented by Geoff Huston at NZNOG 2024
On Starlink, presented by Geoff Huston at NZNOG 2024On Starlink, presented by Geoff Huston at NZNOG 2024
On Starlink, presented by Geoff Huston at NZNOG 2024APNIC
 
Call Girls In Ashram Chowk Delhi 💯Call Us 🔝8264348440🔝
Call Girls In Ashram Chowk Delhi 💯Call Us 🔝8264348440🔝Call Girls In Ashram Chowk Delhi 💯Call Us 🔝8264348440🔝
Call Girls In Ashram Chowk Delhi 💯Call Us 🔝8264348440🔝soniya singh
 
Moving Beyond Twitter/X and Facebook - Social Media for local news providers
Moving Beyond Twitter/X and Facebook - Social Media for local news providersMoving Beyond Twitter/X and Facebook - Social Media for local news providers
Moving Beyond Twitter/X and Facebook - Social Media for local news providersDamian Radcliffe
 
Call Girls Dubai Prolapsed O525547819 Call Girls In Dubai Princes$
Call Girls Dubai Prolapsed O525547819 Call Girls In Dubai Princes$Call Girls Dubai Prolapsed O525547819 Call Girls In Dubai Princes$
Call Girls Dubai Prolapsed O525547819 Call Girls In Dubai Princes$kojalkojal131
 
VIP Kolkata Call Girl Alambazar 👉 8250192130 Available With Room
VIP Kolkata Call Girl Alambazar 👉 8250192130  Available With RoomVIP Kolkata Call Girl Alambazar 👉 8250192130  Available With Room
VIP Kolkata Call Girl Alambazar 👉 8250192130 Available With Roomdivyansh0kumar0
 
Russian Call Girls in Kolkata Ishita 🤌 8250192130 🚀 Vip Call Girls Kolkata
Russian Call Girls in Kolkata Ishita 🤌  8250192130 🚀 Vip Call Girls KolkataRussian Call Girls in Kolkata Ishita 🤌  8250192130 🚀 Vip Call Girls Kolkata
Russian Call Girls in Kolkata Ishita 🤌 8250192130 🚀 Vip Call Girls Kolkataanamikaraghav4
 
Call Girls In Saket Delhi 💯Call Us 🔝8264348440🔝
Call Girls In Saket Delhi 💯Call Us 🔝8264348440🔝Call Girls In Saket Delhi 💯Call Us 🔝8264348440🔝
Call Girls In Saket Delhi 💯Call Us 🔝8264348440🔝soniya singh
 
VIP Call Girls Kolkata Ananya 🤌 8250192130 🚀 Vip Call Girls Kolkata
VIP Call Girls Kolkata Ananya 🤌  8250192130 🚀 Vip Call Girls KolkataVIP Call Girls Kolkata Ananya 🤌  8250192130 🚀 Vip Call Girls Kolkata
VIP Call Girls Kolkata Ananya 🤌 8250192130 🚀 Vip Call Girls Kolkataanamikaraghav4
 
Low Rate Call Girls Kolkata Avani 🤌 8250192130 🚀 Vip Call Girls Kolkata
Low Rate Call Girls Kolkata Avani 🤌  8250192130 🚀 Vip Call Girls KolkataLow Rate Call Girls Kolkata Avani 🤌  8250192130 🚀 Vip Call Girls Kolkata
Low Rate Call Girls Kolkata Avani 🤌 8250192130 🚀 Vip Call Girls Kolkataanamikaraghav4
 
Call Girls In Sukhdev Vihar Delhi 💯Call Us 🔝8264348440🔝
Call Girls In Sukhdev Vihar Delhi 💯Call Us 🔝8264348440🔝Call Girls In Sukhdev Vihar Delhi 💯Call Us 🔝8264348440🔝
Call Girls In Sukhdev Vihar Delhi 💯Call Us 🔝8264348440🔝soniya singh
 
Networking in the Penumbra presented by Geoff Huston at NZNOG
Networking in the Penumbra presented by Geoff Huston at NZNOGNetworking in the Penumbra presented by Geoff Huston at NZNOG
Networking in the Penumbra presented by Geoff Huston at NZNOGAPNIC
 
VIP 7001035870 Find & Meet Hyderabad Call Girls LB Nagar high-profile Call Girl
VIP 7001035870 Find & Meet Hyderabad Call Girls LB Nagar high-profile Call GirlVIP 7001035870 Find & Meet Hyderabad Call Girls LB Nagar high-profile Call Girl
VIP 7001035870 Find & Meet Hyderabad Call Girls LB Nagar high-profile Call Girladitipandeya
 
'Future Evolution of the Internet' delivered by Geoff Huston at Everything Op...
'Future Evolution of the Internet' delivered by Geoff Huston at Everything Op...'Future Evolution of the Internet' delivered by Geoff Huston at Everything Op...
'Future Evolution of the Internet' delivered by Geoff Huston at Everything Op...APNIC
 
Call Girls Service Chandigarh Lucky ❤️ 7710465962 Independent Call Girls In C...
Call Girls Service Chandigarh Lucky ❤️ 7710465962 Independent Call Girls In C...Call Girls Service Chandigarh Lucky ❤️ 7710465962 Independent Call Girls In C...
Call Girls Service Chandigarh Lucky ❤️ 7710465962 Independent Call Girls In C...Sheetaleventcompany
 
Russian Call Girls in Kolkata Samaira 🤌 8250192130 🚀 Vip Call Girls Kolkata
Russian Call Girls in Kolkata Samaira 🤌  8250192130 🚀 Vip Call Girls KolkataRussian Call Girls in Kolkata Samaira 🤌  8250192130 🚀 Vip Call Girls Kolkata
Russian Call Girls in Kolkata Samaira 🤌 8250192130 🚀 Vip Call Girls Kolkataanamikaraghav4
 
VIP Kolkata Call Girl Kestopur 👉 8250192130 Available With Room
VIP Kolkata Call Girl Kestopur 👉 8250192130  Available With RoomVIP Kolkata Call Girl Kestopur 👉 8250192130  Available With Room
VIP Kolkata Call Girl Kestopur 👉 8250192130 Available With Roomdivyansh0kumar0
 
Challengers I Told Ya ShirtChallengers I Told Ya Shirt
Challengers I Told Ya ShirtChallengers I Told Ya ShirtChallengers I Told Ya ShirtChallengers I Told Ya Shirt
Challengers I Told Ya ShirtChallengers I Told Ya Shirtrahman018755
 

Recently uploaded (20)

Pune Airport ( Call Girls ) Pune 6297143586 Hot Model With Sexy Bhabi Ready...
Pune Airport ( Call Girls ) Pune  6297143586  Hot Model With Sexy Bhabi Ready...Pune Airport ( Call Girls ) Pune  6297143586  Hot Model With Sexy Bhabi Ready...
Pune Airport ( Call Girls ) Pune 6297143586 Hot Model With Sexy Bhabi Ready...
 
On Starlink, presented by Geoff Huston at NZNOG 2024
On Starlink, presented by Geoff Huston at NZNOG 2024On Starlink, presented by Geoff Huston at NZNOG 2024
On Starlink, presented by Geoff Huston at NZNOG 2024
 
Call Girls In Ashram Chowk Delhi 💯Call Us 🔝8264348440🔝
Call Girls In Ashram Chowk Delhi 💯Call Us 🔝8264348440🔝Call Girls In Ashram Chowk Delhi 💯Call Us 🔝8264348440🔝
Call Girls In Ashram Chowk Delhi 💯Call Us 🔝8264348440🔝
 
Moving Beyond Twitter/X and Facebook - Social Media for local news providers
Moving Beyond Twitter/X and Facebook - Social Media for local news providersMoving Beyond Twitter/X and Facebook - Social Media for local news providers
Moving Beyond Twitter/X and Facebook - Social Media for local news providers
 
Call Girls Dubai Prolapsed O525547819 Call Girls In Dubai Princes$
Call Girls Dubai Prolapsed O525547819 Call Girls In Dubai Princes$Call Girls Dubai Prolapsed O525547819 Call Girls In Dubai Princes$
Call Girls Dubai Prolapsed O525547819 Call Girls In Dubai Princes$
 
VIP Kolkata Call Girl Alambazar 👉 8250192130 Available With Room
VIP Kolkata Call Girl Alambazar 👉 8250192130  Available With RoomVIP Kolkata Call Girl Alambazar 👉 8250192130  Available With Room
VIP Kolkata Call Girl Alambazar 👉 8250192130 Available With Room
 
Russian Call Girls in Kolkata Ishita 🤌 8250192130 🚀 Vip Call Girls Kolkata
Russian Call Girls in Kolkata Ishita 🤌  8250192130 🚀 Vip Call Girls KolkataRussian Call Girls in Kolkata Ishita 🤌  8250192130 🚀 Vip Call Girls Kolkata
Russian Call Girls in Kolkata Ishita 🤌 8250192130 🚀 Vip Call Girls Kolkata
 
Call Girls In Saket Delhi 💯Call Us 🔝8264348440🔝
Call Girls In Saket Delhi 💯Call Us 🔝8264348440🔝Call Girls In Saket Delhi 💯Call Us 🔝8264348440🔝
Call Girls In Saket Delhi 💯Call Us 🔝8264348440🔝
 
VIP Call Girls Kolkata Ananya 🤌 8250192130 🚀 Vip Call Girls Kolkata
VIP Call Girls Kolkata Ananya 🤌  8250192130 🚀 Vip Call Girls KolkataVIP Call Girls Kolkata Ananya 🤌  8250192130 🚀 Vip Call Girls Kolkata
VIP Call Girls Kolkata Ananya 🤌 8250192130 🚀 Vip Call Girls Kolkata
 
Dwarka Sector 26 Call Girls | Delhi | 9999965857 🫦 Vanshika Verma More Our Se...
Dwarka Sector 26 Call Girls | Delhi | 9999965857 🫦 Vanshika Verma More Our Se...Dwarka Sector 26 Call Girls | Delhi | 9999965857 🫦 Vanshika Verma More Our Se...
Dwarka Sector 26 Call Girls | Delhi | 9999965857 🫦 Vanshika Verma More Our Se...
 
Low Rate Call Girls Kolkata Avani 🤌 8250192130 🚀 Vip Call Girls Kolkata
Low Rate Call Girls Kolkata Avani 🤌  8250192130 🚀 Vip Call Girls KolkataLow Rate Call Girls Kolkata Avani 🤌  8250192130 🚀 Vip Call Girls Kolkata
Low Rate Call Girls Kolkata Avani 🤌 8250192130 🚀 Vip Call Girls Kolkata
 
Call Girls In Sukhdev Vihar Delhi 💯Call Us 🔝8264348440🔝
Call Girls In Sukhdev Vihar Delhi 💯Call Us 🔝8264348440🔝Call Girls In Sukhdev Vihar Delhi 💯Call Us 🔝8264348440🔝
Call Girls In Sukhdev Vihar Delhi 💯Call Us 🔝8264348440🔝
 
Rohini Sector 26 Call Girls Delhi 9999965857 @Sabina Saikh No Advance
Rohini Sector 26 Call Girls Delhi 9999965857 @Sabina Saikh No AdvanceRohini Sector 26 Call Girls Delhi 9999965857 @Sabina Saikh No Advance
Rohini Sector 26 Call Girls Delhi 9999965857 @Sabina Saikh No Advance
 
Networking in the Penumbra presented by Geoff Huston at NZNOG
Networking in the Penumbra presented by Geoff Huston at NZNOGNetworking in the Penumbra presented by Geoff Huston at NZNOG
Networking in the Penumbra presented by Geoff Huston at NZNOG
 
VIP 7001035870 Find & Meet Hyderabad Call Girls LB Nagar high-profile Call Girl
VIP 7001035870 Find & Meet Hyderabad Call Girls LB Nagar high-profile Call GirlVIP 7001035870 Find & Meet Hyderabad Call Girls LB Nagar high-profile Call Girl
VIP 7001035870 Find & Meet Hyderabad Call Girls LB Nagar high-profile Call Girl
 
'Future Evolution of the Internet' delivered by Geoff Huston at Everything Op...
'Future Evolution of the Internet' delivered by Geoff Huston at Everything Op...'Future Evolution of the Internet' delivered by Geoff Huston at Everything Op...
'Future Evolution of the Internet' delivered by Geoff Huston at Everything Op...
 
Call Girls Service Chandigarh Lucky ❤️ 7710465962 Independent Call Girls In C...
Call Girls Service Chandigarh Lucky ❤️ 7710465962 Independent Call Girls In C...Call Girls Service Chandigarh Lucky ❤️ 7710465962 Independent Call Girls In C...
Call Girls Service Chandigarh Lucky ❤️ 7710465962 Independent Call Girls In C...
 
Russian Call Girls in Kolkata Samaira 🤌 8250192130 🚀 Vip Call Girls Kolkata
Russian Call Girls in Kolkata Samaira 🤌  8250192130 🚀 Vip Call Girls KolkataRussian Call Girls in Kolkata Samaira 🤌  8250192130 🚀 Vip Call Girls Kolkata
Russian Call Girls in Kolkata Samaira 🤌 8250192130 🚀 Vip Call Girls Kolkata
 
VIP Kolkata Call Girl Kestopur 👉 8250192130 Available With Room
VIP Kolkata Call Girl Kestopur 👉 8250192130  Available With RoomVIP Kolkata Call Girl Kestopur 👉 8250192130  Available With Room
VIP Kolkata Call Girl Kestopur 👉 8250192130 Available With Room
 
Challengers I Told Ya ShirtChallengers I Told Ya Shirt
Challengers I Told Ya ShirtChallengers I Told Ya ShirtChallengers I Told Ya ShirtChallengers I Told Ya Shirt
Challengers I Told Ya ShirtChallengers I Told Ya Shirt
 

krok presentation medical. dasaaaaaassspptx

  • 1. Krok 1 For medical students III course
  • 2. • An electron micrograph shows a cell-to-cell adhesion consisting, in each cell, of an attachment plaque. The intercellular space is filled with electron-dense substance including transmembrane fibril structures. Specify this adhesion:
  • 3. • A *Desmosome • B Synapse • C Tight junction • D Nexus • E Adherens junction
  • 4. • The action of microbial toxins on the cells caused significant damage of the glycocalyx. What function of the cell membrane will be significantly disordered?
  • 5. • A *Receptor • B Transport • C Respiratory • D Formation of contact • E Protective
  • 6. • The secretory granules appear and disappear in the cytoplasm of pancreatic cells during the secretory cycle. Which structural elements these granules can be included?
  • 7. • A *To inclusions • B To microfilaments • C To lysosomes • D To exocytosis vacuoles • E To granular endoplasmic reticulum
  • 8. • Significant reduction of the protein synthesis in hepatocytes resulted by the long-term effects of the toxic substances on the body. What organelles are most affected by intoxication?
  • 9. • A *Granular endoplasmic reticulum • B Mitochondria • C Microtubules • D Lysosomes • E Golgi apparatus
  • 10. • In the tissue culture the nucleoluses were damaged by the radioactive irradiation. Recovery of what organelles in the cytoplasm becomes problematic?
  • 11. • A *Ribosomes • B Lysosomes • C Endoplasmic reticulum • D Microtubule • E Golgi apparatus
  • 12. • The structure of the ribosome was disordered in the cells. What process is primarily affected?
  • 13. • A *Protein synthesis (translation) • B Protein synthesis (transcription) • C Synthesis of carbohydrates • D Synthesis of lipids • E Synthesis of minerals
  • 14. • The researchers destroyed the structure of one of the cell parts in a scientific experiment. As result the cell lost the ability to division. What structure was broken?
  • 15. • A *Centrosome • B Glycocalix • C Plastic complex • D Microfibrille • E Mitochondria
  • 16. • The patient was hospitalized in the hospital with poisoning. It was established that the detoxification processes were disordered in the liver. Which organelles of hepatocytes were injured?
  • 17. • A *Agranular endoplasmic reticulum • B Mitochondria • C Granular endoplasmic reticulum • D Golgi apparatus • E Ribosomes
  • 18. • The organelles, which consist of cisterns that flattened in the center and extended on the periphery and small vesicles, were founded at the electron microphotography of the nervous cells. What are these organelles?
  • 19. • A *Golgi apparatus • B Centrioles • C Lysosomes • D Peroxisomes • E Mitochondria
  • 20. • The abnormal biopolymers were founded in the body cells of the child (7 years) with congenital "storage diseases". What is the kind of organelles?
  • 21. • A *Lysosomes • B Ribosomes • C Granular endoplasmic reticulum • D Mitochondria • E Peroxisomes
  • 22. • A high content of hydrolytic enzymes in the cytoplasm was founded during the examination. Which organelle activity shows this fact?
  • 23. • A *Lysosomes • B Mitochondria • C Polysomes • D Endoplasmic reticulum • E Centrioles
  • 24. • The organelle that is the big polyprotease complex and consists of tubular and two regulatory parts that are located at both ends of organelle was representing on electronic photo. The function of this organelle is proteolysis. Name this organelle. •
  • 25. • A *Proteasome • B Centrioles • C Inclusion • D Ribosome • E Golgi complex
  • 26. • The vesicles with peroxide oxidation enzymes - catalase, peroxidase (0,05-1,5 microns in diameter) were revealed in the cytoplasm of hepatocytes during the histochemical investigation. What are these organelles?
  • 27. • A * Peroxisomes • B Lysosomes • C Melanosomes • D Liposomes • E Phagosomes
  • 28. • Low level of albumins and fibrinogen was detected in the patient's blood. Decreased activity of what organelle of the liver hepatocytes can cause it?
  • 29. • A *Rough endoplasmic reticulum • B Smooth endoplasmic reticulum • C Mitochondria • D Golgi complex • E Lysosomes • •
  • 30. • There is a large quantity of carbohydrates in the dietary intake of a human. What structures will be seen in the cytoplasm of hepatocytes?
  • 31. • *Glycogen granules • Lipid droplets • One large lipid drop • Lipofuscin inclusions • Increasing of ribosome quantity
  • 32. • At what component of the cell the lysosomes formation takes place?
  • 33. • *At the Golgi apparatus • At the nucleus • At the ribosomes • At the mitochondria • At the organizing cell center
  • 34. • The microtubules contain the protein:
  • 35. • *Tubulin • Desmin • Dynein • Calmodulin • Vimentin
  • 36. • Labeled aminoacids alanine and tryptophane were introducted to a mouse in order to study localization of protein biosynthesis in its cells. Around what organellas will the accumulation of labeled aminoacids be observed?
  • 37. • A *Ribosomes • B Agranular endoplasmic reticulum • C Cell centre • D Lysosomes • E Golgi apparatus
  • 38. • Ultramicroscopic examination of "dark" hepatocytes population in the cell cytoplasm detected a developed granular endoplasmic reticulum. What function has this organelle in these cells?
  • 39. • A *Synthesis of blood plasma proteins • B Carbohydrate synthesis • C Detoxification • D Bile production • E Calcium ion depositing
  • 40. • The hysterectomy was made to a woman 67 years old because of a tumor. At the histological investigation of this tumor the multipolar mitoses were founded at the tumor cells (chromosome disjunction to more than two cellular poles). Disorder of what chromosoms resulted formation of the multipolar mitosis?
  • 41. • A *Centriolies • B Secondary lysosoms • C Smooth endoplasmic reticulum • D Rough endoplasmic reticulum • E Peroxisoms
  • 42. • The cell was treating by a substance, which blocks the nucleotide phosphorylation process in the mitochondria. What process will be broken?
  • 43. • A. *ATP resynthesize • B. Synthesis of mitochondrial proteins • C. Oxidative phosphorylation • D. Integration of functional protein molecules • E. Fragmentation of large mitochondria smaller
  • 44. • The cell of the laboratory animal was overdosed with Roentgen rays. As a result albuminous fragments formed in the cytoplasm. What cell organoid will take part at their utilization?
  • 45. • A *Lysosomes • B Endoplasmic reticulum • C Cells centre • D Golgi complex • E Ribosome •
  • 46. • The histone proteins synthesis was artificially blocked in the cell. What cell structure will be damaged?
  • 47. • A *Nuclear chromatin • B Nucleolus • C Golgi apparatus • D Cell membrane • E Nuclear envelope
  • 48. • The cell without the nucleus and nucleolus is presented at the electron microphotography. The chromosomes are free, the centrioles migrate to the poles. In which phase of the cell cycle is the cell?
  • 49. • A *In prophase • B In anaphase • C In metaphase • D In telophase • E In interphase
  • 50. • 2012 • • The culture of the tumor cells was affected by colchicine, which inhibits formation of the proteins-tubulins that are necessary for the spindle apparatus formation. What stage of the cell cycle will be affected?
  • 51. • A *Mitosis • B Presynthetic period • C Synthetic period • D Postsynthetic period • E G0 - phase •
  • 52. • A human somatic cells in the metaphase mitosis was founded in the histological preparation. How many chromosomes the metaphase plate consist of in case if each chromosome has two sister chromatids.
  • 53. • A *46 chromosomes • B 92 chromosomes • C 23 chromosomes • D 48 chromosomes • E 24 chromosomes
  • 54. • • The cells whose nuclei contain sex chromatin (Barr corpuscle) were founded in the amniotic fluid during the investigation that obtained by amniocentesis (amniotic membrane puncture). What could it mean?
  • 55. • A *Development of the female sex fetus • B Development of the male sex fetus • C Genetic abnormalities in fetal development • D Trisomy • E Рolyploidy •
  • 56. • It is known that the synthesis of the proteins- tubulins is blockade under the colchicine action. What stage of the cell cycle will be disordered in this case?
  • 57. • *Postsynthetic period of the interphase • Prophase of mitosis • Synthetic period of the interphase • Formation of the metaphase plate • Presynthetic period of the interphase
  • 58. • A very low content of heterochromatin at the hepatocyte nuclei founded during microscopic study. What functional state of the cell does it mean?
  • 59. • A *Increase of protein synthesis • B Apoptosis • C Necrosis • D Reduction of protein synthesis • E Entry into mitosis •
  • 60. • Compaction of the nucleus was observed on the electron microphotography of the cells taken from animals after a chemical exposure. What is the state of the nucleus in the cell?
  • 61. • A *Karyopyknosis • B Mitosis • C Amitosis • D Polyploidy • E Meiosis
  • 62. • At the electronic microphotography the nucleus is surrounded by the thin lamina of biological membrane. What is it name?
  • 63. • A * Nuclear envelope • B Plasmolemma • C Cytolemma • D Nuclear pore • E Pore complex
  • 64. • The navy-blue chromatin granules were founded at the nucleus on the slide that is stained by hematoxylin and eosin. What is the stage of the cell cycle?
  • 65. • A * Interphase • B Prophase • C Metaphase • D Anaphase • E Telophase • •
  • 66. • While studying maximally spiraled chromosomes of human karyotype the process of cell division was stopped in the following phase:
  • 67. • A *Metaphase • B Prophase • C Interphase • D Anaphase • E Telophase
  • 68. • Moving of the daughter chromatids to the poles of the cell is observed in the mitotically dividing cell. On what stage of the mitotic cycle is this cell?
  • 69. • A *Anaphase • B Metaphase • C Telophase • D Prophase • E Interphase
  • 70. • The study of mitotic cycle phases of onion root revealed the cell, in which the chromosomes are situated in the equatorial plane, forming a star. What stage of the cell mitosis is it?
  • 71. • A *Metaphase • B Anaphase • C Telophase • D Interphase • E Prophase
  • 72. • During the postsynthetic period of mitotic cycle the synthesis of proteins - tubulins, which take part in the mitosis formation, was destroyed. It can cause the impairment of:
  • 73. • A *Chromosome separation • B Cytokinesis • C Duration of mitosis • D Chromosome despiralization • E Chromosome spiralization
  • 74. • There are three periods of the cell cycle interphase. What process is take place during the S-period?
  • 75. • A *Replication of the DNA • B Meiosis • C Cytokinesis • D Mitosis • E Аmitosis
  • 76. • EMBRYOLOGY. COMPARATIVE EMBRYOLOGY OF BIRDS AND MAMMALS •
  • 77. • The chicken embryo at the stage of the mesoderm differentiation to somites and splanchnotom was revealed in the histological preparation. What material is the axial skeleton developing from?
  • 78. • A *Sclerotome • B Dermatome • C Nephrotome • D Splanchnotom • E Myotome
  • 79. • The gametes precursors (gonoblasts) were revealed in the embryo at 2nd - 3rd weeks of embryogenesis. Where do these cells differentiate?
  • 80. • A *In the yolk sac • B In the mesenchyme • C In the embryonic ectoderm • D In dermatomes • E In the embryonic endoderm
  • 81. • Study of the biopsy material of an embryo revealed a zone of developmental abnormality in a somite. The zone was located close to the endoderm and the notochord. What formations may have abnormal development in case of pregnancy continuation?
  • 82. • A *Skeletal tissues • B Genito-urinary system • C Skeletal striated muscle tissue • D Cardiac striated muscle tissue • E Fibrous connective tissue of the skin
  • 83. • The sclerotome was destroyed in the bird embryo during the experiment. Disorder of what structure can be caused by this manipulation?
  • 84. • A * Axial skeleton • B Skin connective tissue • C Internal organs stroma • D Gonadal stroma • E Chord
  • 85. • The myotome was destroyed in the rabbit embryo during the experiment. Disorder of what structure can be caused by this manipulation?
  • 86. • A * Skeletal muscle • B Axial skeleton • C Skin connective tissue • D Smooth muscle • E Serous membranes
  • 87. • The outer germ layer (ectoderm) was destroyed in the frog embryo during the experiment. What morphological structure of this embryo will be not developed?
  • 88. • A * Epidermis • B Somites • C Nephrotome • D Splanchnotom • E Bone tissue
  • 89. • The embryo on the stage of early gastrulation was founded during the forensic medical expertise of a woman who died in the road accident. Name the place of the embryo localization in case of its normal development.
  • 90. • A *Uterus wall • B Ampullar part of the oviduct • C Uterus part of the oviduct • D Ovary • E Abdominal cavity •
  • 91. • The embryo composed by to blastomeres was founded during the microscopic investigation of female reproductive organs removed during the operation. Name the place of the embryo localization in case of its normal development.
  • 92. • A * Oviduct, close to its ampullar part • B Oviduct, close to its uterus part • C Uterus cavity • D Abdominal cavity • E Ovary
  • 93. • The process of a zygote cleavage ends with the blastula formation. What type of blastula is typical for a human?
  • 94. • A * Blastocyst • B Coeloblastula • C Discoblastula • D Amphiblastula • E Morula
  • 95. • The human embryo that is not attached to the endometrium was found in the uterus cavity. What is the stage of embryogenesis?
  • 96. • A * Blastocyst • B Zygote • C Morula • D Gastrula • E Neurula
  • 97. • The human ovum represented on histological preparation. There are small amount of yolk inclusions in the cytoplasm of it. Identify the type of ovum.
  • 98. • A *Secondary isolecithal • B Isolecithal • C Telolecithal • D Alecithal • E Centrolecital •
  • 99. • Chronic poisoning of 38-year-old miner by cadmium compounds resulted in male sterility. Semen analysis revealed the inability of sperm to move. Damage of what cytoskeleton components may be responsible for this pathology?
  • 100. • A *Axoneme microtubules • B Actin microfilaments • C Intermediate filaments • D Microtubule spindle • E Actomyosine complex •
  • 101. • The primary Hensen's node hasn’t formed in the embryo during the gastrulation. The development of what axial organ will be inhibiting?
  • 102. • A * Chorda • B Neural crest • C Neural groove • D Neural tube • E Mantle layer of the neural tube •
  • 103. • 2013 • • The human blastocyst implantation begins. What is the period of embryogenesis, which starts at the same time with implantation?
  • 104. • A * Gastrulation • B Invagination • C Differentiation • D Histogenesis • E Cleavage •
  • 105. • The chorion is determined at the microscopic examination of the embryo membranes. What is the main function of this organ?
  • 106. • A * Exchange of substances between the mother and fetus. • B Hematopoietic. • C Production of the amniotic fluid. • D Formation of the primordial germ cells. • E Formation of the lymphocytes
  • 107. • The early gastrulation of the human embryo occurs by delamination of the embryoblast. At what structure does the nervous system rudiment dispose?
  • 108. • A * At the epiblast • B At the trophoblast • C At the hypoblast • D At the marginal zone of the hypoblast • E At the central zone of the hypoblast
  • 109. • The hydramnios was diagnosed at the pregnant women during the ultrasound examination. Disorder of what extraembryonic organ function can result in such pathological condition?
  • 110. • A *Amniotic membrane • B Chorion • C Placenta • D Yolk sac • E Allantois
  • 111. • The embryonic shield with two layers of cells (ectoderm and entoderm) was revealed in the human embryo taken from spontaneous abortion. At what stage of embryonic development was an embryo?
  • 112. • A * Gastrulation • B Cleavage • C Progenesis • D Organogenesis • E -
  • 113. • It is known that the megalocytes may appear in the human peripheral blood. When the appearance of these cells in the blood is normal?
  • 114. • A * In the embryonic period • B At the age of 1 year • C At the age from 1 to 30 years • D In old age • E During the pregnancy
  • 115. • The human embryo implantation in the uterine wall (during the 7th day) is the one of the critical periods of embryogenesis. What gastrulation process is occurs in embryoblast during this period?
  • 116. • A * Delamination • B Migration • C Epiboly • D Invagination • E Neurulation •
  • 117. • "A person was born in a shirt." What kind of "shirt" is referred in this proverb about?
  • 118. • A * Amniotic • B Yolk • C Serous • D Chorionic • E Trophoblastic
  • 119. • • 2011-2005 • • Disorder of the cerebrum development was founded during the ultrasonic scanning of the pregnant woman. The chronic alcoholism is in anamnesis. To what critical period of the embriogenesis this pathology is conform?
  • 120. • *15-20 weeks of embriogenesis • 20-24 weeks of embriogenesis • 7-8 days of embriogenesis • 3-8 weeks of embriogenesis • Neonatal period •
  • 121. • The mesenchyme at the yolk-sac wall was destroyed on the early stage of embryogenesis. What consequences can develop as result of this manipulation?
  • 122. • *Disorder of the blood vessels formation • Disorder of embryo trophisity • Disorder of metabolic products excretion • Disorder of the allantois formation • Disorder of the amnion formation
  • 123. In course of a conditional experiment, the development of mesenchyme cells was completely inhibited. Development of the following muscular tissue will be disturbed:
  • 124. • A *Smooth muscle tissue • B Neural muscle tissue • C Epidermal muscle tissue • D Cardiac muscle tissue • E Skeletal muscle tissue
  • 126. • The villus of the small intestine is covered by the tissue that consist only of the cells, which form a layer on the basement membrane. The tissue does not contain blood vessels. What tissue covers the surface of the villus?
  • 127. • A * Epithelial tissue • B Cemented irregular fibrous connective tissue • C Dense irregular connective tissue • D Smooth muscle tissue • E Reticular tissue •
  • 128. • The structures of tight junction between the epithelial cells were affected during the experiment. What function of the epithelium will be disordered?
  • 129. • A * Mechanical • B Absorption • C Vitamin D-producing • D Secretory • E Excretory
  • 130. • Microslide contains the preparation of a gland composed of several secretory saccule-shaped parts that open in the common excretory duct. What gland is it?
  • 131. • A *Simple branched alveolar gland • B Compound branched alveolar gland • C Simple unbranched alveolar gland • D Compound unbranched alveolar gland • E Simple branched tubular gland •
  • 132. • The secretory parts of the apocrine glands contain myoepithelial cells/ what is the function of these cells?
  • 133. • A *Contractive • B Secretory • C Protective • D Regenerative • E Supporting •
  • 134. • A scheme presents an exocrine gland that has unbranched excretory duct with a terminal part in form of a saccule opening into the duct. How is this gland called according to the morphological classification of exocrine glands?
  • 135. • A *Simple unbranched alveolar • B Compound branched alveolar • C Simple branched tubular • D Compound unbranched alveolar • E Compound unbranched alveolar tubular •
  • 136. • The patient is complaining of the black spots appearance on the face. Upon examination, it was founded that the appearance of these spots is associated with the violation of the sebaceous glands secretion. What type of secretion is characteristic for these glands?
  • 137. • A *Holocrine • B Merocrine • C Macroapocrine • D Microapocrine • E Merocrine and microapocrine •
  • 138. • The gland that consists from two acinary secretory parts are opening into common excretory duct was founded in the skin. What is the kind of a gland?
  • 139. • A *Simple branched alveolar gland • B Compound branched alveolar gland • C Simple unbranched alveolar gland • D Compound unbranched alveolar gland • E Simple branched tubular gland
  • 140. • The patient blood was taken for analysis. The 30% of red blood cells with an irregular shape were founded. Clinically, this phenomenon is described as:
  • 141. • A * Pathological poikylocytosis • B Anisocytosis • C Physiological poikylocytosis • D Macrocytosis • E Microcytosis •
  • 142. • The acute decline of the hemoglobin was revealed in the patient's blood during the examination at the clinic. What is the function of blood will be disordered?
  • 143. • A * Respiratory • B Humoral • C Homeostatic • D Protective • E Trophic •
  • 144. • The anemia developed at the patient 50 years old with chronic nephritis. What was the most likely cause of the anemia at this patient?
  • 145. • A * Decreasing of erythropoietin production • B Absence of gland • C Lack of vitamin B12 • D Disorders of porphyrin synthesis • E Immunological damage cells - precursors of erythropoiesis •
  • 146. • In the red bone marrow the blood cells, which develop, are located by the islands. Some of the islands associated with macrophages. What blood cells are developed in these islands?
  • 147. • A * Erythrocytes • B Precursors of T- and B-lymphocytes • C Monocytes • D Platelets • E Basophilic granulocytes •
  • 148. • The reduced hemoglobin amount was revealed in the blood test. What function of the blood will be disordered?
  • 149. • A * Transport of gases • B Transport of hormones • C Providing immunity • D Clotting • E Transport of nutrients
  • 150. • In the blood of a 26-year-old man it was revealed 18% of erythrocytes of the spherical, ball- shaped, flat and thorn-like shape. Other erythrocytes were in the form of the concavo- concave disks. How is such phenomenon called?
  • 151. • A *Physiological poikylocytosis • B Pathological poikylocytosis • C Physiological anisocytosis • D Pathological anisocytosis • E Erytrocytosis
  • 152. • The 12,5% erythrocytes with diameter greater than 8 microns and 12.5% erythrocytes with diameter less than 6 microns were revealed in the patient's blood. The rest of red blood cells have a diameter of 7,1–7,9 microns. What is the name of this phenomenon?
  • 153. • A * Physiological anisocytosis • B Pathological anisocytosis • C Physiological poikilocytosis • D Pathological poikilocytosis • E Erythrocytosis •
  • 154. • 2001-2011 • • Transfusion of Rh-positive blood to Rh-negative patient results in the formation of Rh antibodies and hemolysis. What blood cells are the carrier of Rh factor? •
  • 155. • A *Erythrocytes • B Platelets • C Lymphocytes • D Monocytes • E Neutrophils •
  • 156. • The signs of inflammation: pain, redness, edema as signs of immediate hypersensitivity were revealed at the child around the skin wound. Which blood cells are causing these changes?
  • 157. • A * Basophils • B Eosinophils • C Neutrophils • D Lymphocytes • E Monocytes •
  • 158. • According to results of the blood stains studying at the crime scene the forensic pathologist determined that it was female blood. What signs were on the base of this conclusion?
  • 159. • A * Presence of the satellite in the neutrophils nuclei • B Presence of the microcytes and macrocytes • C Poikilocytosis • D Presence of the eosinophils specific granules • E According to the number of red blood cells
  • 160. • The cells with the histamine and heparin granules in the cytoplasm were defined in the blood smear. What is the kind of the cell?
  • 161. • A * Basophils • B Neutrophils • C Eosinophils • D Monocytes • E Erythrocytes • •
  • 162. • The cells, which accounts for 0,5% of the total leukocytes number with the S-shaped curved nucleus and metachromatic colored granules in the cytoplasm, were founded in the patient's blood smear. What are these cells?
  • 163. • A * Basophils • B Neutrophils • C Eosinophils • D Monocytes • E Lymphocytes •
  • 164. • The numerous plasma cells plasmocytes were revealed in the blood of a 16 years old girl with autoimmune inflammation of the thyroid gland. With the proliferation and differentiation of what blood cells the increase of the plasmocytes number is associated?
  • 165. • A * B-lymphocytes • B T-helper • C Tissue basophils • D T-killer • E T-suppressor •
  • 166. • The B-lymphocytes were marked with the tracer in the experiment. The foreign protein was injected under the skin of the animal. Which cells in the connective tissue will contain this tracer?
  • 167. • A * Plasmocytes • B T-lymphocytes • C Macrophages • D Tissue basophils • E Fibroblasts •
  • 168. • One of the blood cells populations was selectively stimulated in the experiment. Permeability of blood vessels was significantly increased as result of it. That leads to edema of perivascular tissue development end deceleration of the blood clotting. Which blood cells were stimulated?
  • 169. • A * Basophils • B Erythrocytes • C Platelets • D Eosinophils • E Lymphocytes
  • 170. • The rounded cells with the segmented nuclei are predominating from the leukocytes in the smear of peripheral blood. The granules of their cytoplasm stained both acidic and basic dyes. What are these cells?
  • 171. • A * Segmented neutrophils • B Basophils • C Eosinophils • D Young neutrophils • E Monocytes •
  • 172. • The large cells with low-basophilic cytoplasm and bean-shaped nucleus were founded in a blood smear. The cell is the largest from visible in the visual field. What are the cells?
  • 173. • A * Monocytes • B Macrophages • C Plasmocytes • D Middle lymphocytes • E Small lymphocytes •
  • 174. • 2013 • • To determine the functional activity of blood cells the suspension of the microorganisms was introduced into a test tube containing leukocyte mass. Inside of what cells the phagocytized bacteria will be detected?
  • 175. • A * Neutrophils and monocytes • B Lymphocytes and basophils • C Lymphocytes and eosinophils • D Monocytes and lymphocytes • E Lymphocytes and neutrophils •
  • 176. • The neutrophils were defined during the study of the connective tissue microslide. What is the function of these cells in the tissues?
  • 177. • A * Phagocytosis of microorganisms • B Trophic • C Support function • D Regulation of sthe mooth muscle cells contraction • E Dilatation of the blood vessels. •
  • 178. • It is known that plasmocytes produce specific antibodies against of the antigen. The number of plasmocytes is increases after the antigen introduction. At the expense of which blood cells the plasmocytes number will increase?
  • 179. • A * B-lymphocytes • B T-lymphocytes • C Monocytes • D Basophils • E Eosinophil •
  • 180. • 2012 • • The increasing of the total leukocytes number was revealed in the general blood analysis of a patient with pneumonia. What is the name of this phenomenon?
  • 181. • A * Leukocytosis • B Anemia • C Leukopenia • D Anisocytosis • E Poikilocytosis
  • 182. • A live vaccine injected into a human organism. Increased activity of what connective tissue cells can be expected?
  • 183. • A *Plasma cells and lymphocytes • B Macrophages and fibroblasts • C Pigmented cells and pericytes • D Adipocytes and adventitial cells • E Fibroblasts and labrocytes •
  • 184. • The chromatin of one of the neutrophils nucleus segments is forming the drumstick. What is the name of this structural formation?
  • 185. • A * Barr's body • B Lyon’s body • C Decondensed chromatin • D Euchromatin • E Pacinian corpuscles •
  • 186. • The helminthiasis was diagnosed at the 6 year child. What changes of the leukogram can be expected.
  • 187. • A * Increasing of the eosinophils number • B Increasing of the neutrophils number • C Reducing the eosinophils number • D Increasing of the monocytes number • E Increasing of the lymphocytes number •
  • 188. • The nurse complains of hands injury that resembls the eczema. She said that after streptomycin injection( she makes it to the patient) the skin itching is increased and the vesicles with the watery fluid appear on the skin. The symptoms disappear during the holidays. The blood test was made on suspicion of allergic reaction. Increasing the number of what blood cells can be detected?
  • 189. • A * Eosinophilic leukocytes • B Basophilic leukocytes • C Monocytes • D Neutrophilic leukocytes • E Lymphocytes •
  • 190. • The 20% large (diameter 20 mm), rounded cells with low-basophilic cytoplasm and bean-shaped nucleus were founded in a blood smear. Clinically, this phenomenon is described as:
  • 191. • A * Monocytosis • B Lymphocytosis • C Leukopenia • D Neutrophilia • E Reticulocytosis •
  • 192. • The cells, which make up 0.5% of the total white blood cells number were revealed at the patient's blood smear. They have S-shaped nucleus and metachromatic colored granules in the cytoplasm. What are these cells?
  • 193. • A *Basophils • B Neutrophils • C Eosinophils • D Monocytes • E Lymphocytes •
  • 194. • The helminthosis was detected at the child (10 years). What changes in the WBC can be expected?
  • 195. • A * Number of eosinophils will increase • B Number of platelets will increase • C Number of red blood cells will increase • D Number of segmented neutrophils will increase • E Number of basophils will increase •
  • 196. • In the analysis investigator made an additional conclusion that the blood belongs to the female. Specific features of what cells structure enable us to conclusion?
  • 197. • A * Neutrophilic leukocytes • B Erythrocytes • C Lymphocytes • D Monocytes • E Basophilic leukocytes
  • 198. • The cells of next morphology: intensely basophilic cytoplasm, eccentrically placed nucleus with chromatin, which is located in a "spoke wheel" and highlights the cytoplasm near it – were founded in the lymph node histological sections of the experimental animals after the antigen stimulation. What is the type of cell?
  • 199. • A * Plasmocytes • B Macrophages • C Fibroblasts • D Adipocytes • E Tissue basophils (mast cells) •
  • 200. • Blood sampling for bulk analysis is recommended to be performed on an empty stomach and in the morning. What changes in blood composition can occur if to perform blood sampling after food intake?
  • 201. • A *Increased contents of leukocytes • B Increased contents of erythrocytes • C Increased plasma proteins • D Reduced contents of plateletes • E Reduced contents of erythrocytes •
  • 202. • The graft rejection took place after the heterotransplantation. Which blood cells provide the process?
  • 203. • A * T-killers • B T-helpers • C T-suppressors • D T0- lymphocytes • E T-memory cells •
  • 204. • The antibodies are producing at the secondary penetration of the antigen into the organism. By the function of what cell does it resulted?
  • 205. • A *Memory cells • B T-killers • C T-suppressors • D Macrophages • E Dendrite cells
  • 206. • A large number of rounded cells with segmented nucleus (three or more segments) and small pink- purple granules in the cytoplasm were reveale in a blood smear of patient with pneumonia. What is a cell?
  • 207. • A *Neutrophils • B Monocytes • C Eosinophilic granulocytes • D Basophilic granulocytes • E Lymphocytes
  • 208. • General blood test of a child found the percentage of lymphocytes and neutrophils in the ratio of 45:45. Which age may correspond to a physiological crossroads?
  • 209. • A * 5 days and 5 years • B 14 years • C 1 day and 3 years • D 1 year • E 1 day
  • 210. • Presence of inflammation in the kidney observed at a patient. Blood test was make. It was observe a large number of cells with following cell morphology: nucleus with 2 - 5 segments, pink – purple granules in the cytoplasm. What is a cell?
  • 211. • A *Neutrophilic granulocytes segmented • B Monocytes • C Lymphocytes • D Acidophilic granulocytes • E Basophilic granulocytes •
  • 212. • A blood test made to a patients with allergic rhinitis. In a blood was observe a large number of cells with following cell morphology: nucleus with 2 - 3 segments, oxyphilic cytoplasm filled with bright pink large granules. What is a cell?
  • 213. • A *Acidophilic granulocytes • B Lymphocytes • C Monocytes • D Basophilic granulocytes • E Neutrophils •
  • 214. • Among of blood cells there are cells whose number is 3-11% of the total number of leukocytes. The main functions of these cells – conversion into a macrophagic system cells; simplest phagocytosis, aged cells phagocytosis. What are these cells?
  • 215. • A *Monocytes • B Basophils • C Eosinophils • D Lymphocytes • E Neutrophils •
  • 216. • A blood test made to a patient with pneumonia. The blood test reveal an increase in the number of cells that have bean-shaped and rod-shaped nucleus, small granules, some of which are dyed basophilic other oxyphilic. Name such changes of WBC.
  • 217. • A. *WBC shift to the left • B. WBC shift to the right • C. Neutrocytopenia • D. Neuthrocytosis • E. Lymphocytosis
  • 218. • Filopodium of megakaryocytes are going through the pores of red bone marrow sinusoidal capillaries into the lumen of blood vessels, where they are fragmenting into individual plates. What blood cells are formed in this way?
  • 219. • A * Platelets • B Erythrocytes • C Lymphocytes • D Reticulocytes • E Monocytes •
  • 220. • In a blood was observe a large number of cells with following cell morphology: nucleus with 2 - 3 segments, a cytoplasm filled with large granules that at staining showing metachromasia. What is the cell?
  • 221. • A. *Basophils • B. Neutrophils • C Eosinophils • D. Monocytes • E. Red blood cells •
  • 222. • One of the blood cells populations was selectively stimulate. As a result of it the process of blood clotting was significantly slowed. Which blood cells subjected to stimulation?
  • 223. • A. *Basophils • B. Platelets • C. Monocytes • D. Eosinophils • E. Neutrophils •
  • 224. • A 16% rounded cell with sizes 4.5 ... 7 mm, having a large spherical nucleus, basophilic cytoplasm painted in a narrow border around the nucleus were detected in a blood smear of a patient after suffering from flu. What state of blood do they describe?
  • 225. • A. * Lymphocytopenia • B. Monocytosis • C. Neutrocytosis • D. Lymphocytosis • E. Monocytopenia • •
  • 226. • The significant reduction of megakaryocytes was founded during the histological investigation of the red bone marrow punctate of 35 years old patient. How will change the correlation of blood cells in this case?
  • 227. • A * Number of platelets will decrease • B Number of red blood cells will decrease • C Number of eosinophils will decrease • D Number of neutrophils will decrease • E Number of B-lymphocytes will decrease •
  • 228. • It is known that megalocytes can be present in the peripheral blood of human. When these cells can be present in the peripheral blood in norm?
  • 229. • A *In the embryogenesis • B Till the age 1 year • C At the age from 1 to 30 • D At the old age • E During the pregnancy
  • 230. • During postembryonic haemopoiesis in the red bone marrow the cells of one of the cellular differon demonstrate a gradual decrease in cytoplasmic basophilia as well as an increase in oxyphilia, the nucleus is being forced out. Such morphological changes are typical for the following haemopoiesis type:
  • 231. • A *Erythropoiesis • B Lymphopoiesis • C Neutrophil cytopoiesis • D Eosinophil cytopoiesis • E Basophil cytopoiesis •
  • 232. • The megakaryocyte with demarcation channels in the peripheral part of the cytoplasm was determined at the electron microphotographs of the red bone marrow. What is the function of these structures?
  • 233. • A * Formation of platelets • B Increasing of the cell surface • C Increase of the ion channels number • D Cell division • E Cell destruction
  • 234. • The cells of granulocytic series were revealed at the biopsy material of red bone marrow. Specify what changes occur in the nucleus during the differentiation of these cells?
  • 235. • A * Segmentation • B Polyploidisation • C Pyknosis • D Enucleation • E Increasing the size
  • 236. • The cells, differentiation of which is characterized by pycnosis and nucleus removing, were revealed in the myeloid tissue punctate of a 6 years old child. What kind of hematopoiesisis characterized by such morphological changes?
  • 237. • A * Erythropoiesis • B Thrombocytopoiesis • C Lymphocytopoiesis • D Granulocytopoiesis • E Monocytopoiesis
  • 238. • A newborn baby has the disorder of the thymus development. What type of the hematopoiesis will be disordered?
  • 239. • A * Lymphocytopoiesis • B Monocytopoiesis • C Erythropoiesis • D Granulocytopoiesis • E Thrombocytopoiesis
  • 240. • After the radioactive exposure a patient has stem cells disorder. The regeneration of what cells of friable connective tissue will be damaged?
  • 241. • A. *Macrophages • B. Pericytes • C. Fibroblasts • D. Pigment cells • E. Adipocytes •
  • 242. • In course of an experiment, a big number of stem cells of red bone marrow was in some way destruct. Regeneration of which cell populations in the loose connective tissue will be inhibit?
  • 243. • A *Of macrophages • B Of fibroblasts • C Of pigment cells • D Of lipocytes • E Of pericytes •
  • 244. • Formation of structurally complex - In the development of red blood cells in lymphoid tissue become - erythroblastic island in the center of which there is a cell, the processes of which encompass the erythroblasts. Identify this cell.
  • 245. • A *Macrophage • B Fibroblasts • C Endothelial • D Lipocyte • E Pericyte •
  • 246. • The least differentiated cells that are forming populations and characterized by self-inducing, differentiation in several ways through the progenitor cells into functionally mature cells, are present at every tissues during the human life (or were present in embryogenesis). What are these cells?
  • 247. • A * Stem cells • B Specialized cells • C Multifunction cells • D Cells of histogenesis • E Blastomere •
  • 248. • The cells that characterized by polychromatophilia, appears of meshwork (remnants of granular endoplasmic reticulum and free ribosomes) in the cytoplasm present in the blood smear. Their number is normally 1-5% of the total number of red blood cells, and increasing their numbers is diagnostic feature of enhanced hematopoiesis. What are these cells?
  • 249. • A * Reticulocytes • B Erythroblasts • C Erythroblasts polychromatophilic • D Basophilic erythroblast • E Proerythroblast
  • 250. • The method of dactylography is used extensively in the forensic medical expertise. This method is based on the strictly individual picture of the skin surface made by the papillary layer of the derma. What tissue is forming this layer of the derma?
  • 251. • A * Loose connective tissue • B Dense irregular fibrous tissue • C Dense regular fibrous tissue • D Reticular tissue • E Adipose tissue • •
  • 252. • The histamine plays a central role in the development of allergic clinical manifestations. What cells produce it?
  • 253. • A * Mast cells • B T lymphocytes • C Macrophages • D B-lymphocytes • E Plasmocytes •
  • 254. • The local esophagus stenosis as a result of scar formation developed at the patient after the esophagus chemical burn. Which cells of the loose connective tissue involved in the formation of scars?
  • 255. • A * Mature specialized fibroblasts • B Young unspecialized fibroblasts • C Fibrocytes • D Myofibroblasts • E Fibroclasts •
  • 256. • 2013 • • A foreign body entered into the skin and leds to inflammation development. What connective tissue cells take place in the skin reaction agains of the foreign body?
  • 257. • A * Neutrophils, macrophages, fibroblasts • B Macrophages • C Melanocytes • D Adipocytes • E Adventicial cells
  • 258. • • Live vaccine is injected into the human body. Increasing activity of what cells of connective tissue can be expected?
  • 259. • A *Plasmocytes and lymphocytes • B Macrophages and fibroblasts • C Pigmentocytes and pericytes • D Adipocytes and adventitious cells • E Fibroblasts and labrocytes •
  • 260. • 2012 • • The connective tissue defects scar is developing in the wound place during the healing. Which cells provide the process?
  • 261. • A * Fibroblasts • B Macrophages • C Fibrocytes • D Mast cells • E Melanocytes •
  • 262. • The allergic dermatitis of both hands developed at the women as a result of the contact with the chromium compounds at the production. Which skin cells mainly participated in the realization of this disease?
  • 263. • A * Tissue basophils • B Plasmocytes • C Macrophages • D Neutrophils • E Lymphocytes
  • 264. • A big number of the elongated cells with dense nucleus and many lysosomes, phagosomes and pinocytotic vacuoles in the basophilic cytoplasm were founded in the lavage of the patient with acute tibia wound. What are these cells?
  • 265. • A * Macrophages of connective tissue • B Fibroblasts • C Fibrocytes • D Plasmocytes • E Tissue basophils •
  • 266. • In course of an experiment a big number of stem cells of red bone marrow was in some way destructed. Regeneration of which cell populations in the loose connective tissue will be inhibited?
  • 267. • A *Of macrophags • B Of fibroblasts • C Of pigment cells • D Of lipocytes • E Of pericytes •
  • 268. • A significant number of red bone marrow stem cells was destroyed at the experiment. Renewal of what cells of connective tissue will be inhibited?
  • 269. • A * Macrophages • B Fibroblasts • C Pigment cells • D Adipocytes • E Pericytes •
  • 270. • An inflammation is characterized by the dilation of the blood capillaries at the site of injury, decreased blood circulation, increased vascular permeability. Which of the following cells play the primary role in this processes?
  • 271. • A * Tissue basophils • B Fibroblasts • C Plasmocytes • D Eosinophils • E Macrophages • •
  • 272. • 2005-2011 • • Low concentration of the specific antibodies was founded in the blood of a patient with the influenza. The fuction of what cell of connective tissue is inhibited?
  • 273. • *Plasmocyte • Lymphocyte • Macrophage • Macrophage • Labrocyte •
  • 274. • The microscopic examination of wound lavage of a patient with acute wound process of his shin revealed big contents of irregular extended- formed cells, with tough nucleus, the basophilic cytoplasm of which includes many lysosomes, phagosomes and pinocytotic bubbles. What cells were find out in the wound?
  • 275. • A. Tissue basophils • B. *Connective tissue macrophages • C. Fibrocytes • D. Fibroblasts • E. Plasmocytes •
  • 276. • A bleeding accompanied by slow blood clotting (at normal number of platelets in blood tests) occurred in women after the limb injury. What substance of loose connective tissue prevents the blood clotting process? By what cell does it produced?
  • 277. • A. * Heparin, tissue basophils • B. Heparin, macrophages • C. Histamine, tissue basophils • D. Heparin, plasma cells • E. Heparin, fibroblast
  • 278. • The allergic reaction in the form of urticaria developed at a patient (formations of the vesicles under the epidermis as result of the plasma release into loose connective tissue). What substance of mast cells causes an increase of blood vessel permeability?
  • 279. • A. * Histamine • B. Heparin • C. Hyaluronic acid • D. Chondroitin sulfate type A • E. Chondroitin sulfate type C
  • 280. • Leading role in the vascular phase of inflammation the histamine plays. Which cell of loose connective tissue produces histamine?
  • 281. • A. * Tissue basophils • B. Fibroblasts • C. Plasma cell • D. Fibrocyte • E. Macrophage •
  • 282. • The drug ketotifen, which is able to inhibit the histamine release from a particular cell type, used for long-term therapy in patients with asthma. Name these cells.
  • 283. • A *Mast cells • B Lymphocytes • C Eosinophils • D Macrophages • E Plasma cells
  • 284. • In allergic rhinitis (inflammation of the nasal mucosa) increasing the number of basophils in the connective tissue of the mucous accompanied by oedema of the tissues. With what function of the tissue basophils does it connected?
  • 285. • A * Biogenic amines synthesis • B Intercellular substance production • C Phagocytosis • D Antibody production • E Heat production • •
  • 286. • The lower limb was injured during the athlete training. Traumatologist made the diagnosis: the tendon rupture. What type of connective tissue the tendon belongs to?
  • 287. • A * Dense regular fibrous tissue • B Dense irregular fibrous tissue • C Loose connective tissue • D Reticular tissue • E Cartilage tissue
  • 288. • The damaged Achilles tendon function was renewed after the treating. What is the mechanism of a tendon regeneration?
  • 289. • A * Synthesis of the collagen fibers • B Synthesis of the hyaline cartilage • C Formation of the adipose tissue • D Synthesis of the fibrous cartilage • E Replacements of the place of injury by the muscle tissue
  • 290. • Decreased blood supply to the organs causes hypoxia that activates fibroblasts function. Volume of what elements is increased in this case?
  • 291. • A *Intercellular substance • B Vessels of microcircular stream • C Nerve elements • D Parenchyme elements of the organ • E Lymphatic vessels
  • 292. • The substance that disorders the collagen fibers formation was introduced to the animals. How will change the tendon properties in this case?
  • 293. • A * The tendon hardness to a rupture will decrease • B No change • C The tendon elasticity will decrease • D The tendon hardness to a rupture and elasticity will decrease • E The tendon hardness to a rupture will increase but elasticity will decrease
  • 294. • The caesarean section was performed to a patient. The uterus wall was cut and the fetus was disengaged. What is the mechanism of uterus healing in the area the myometrium injury?
  • 295. • A * Formation of the connective scar • B Neoplasms of the smooth muscular tissue • C Formation of the striated muscle • D Proliferation of the myosatelits • E Hypertrophy of the smooth muscle cells
  • 296. • Decreased blood supply to the organs causes hypoxia that activates fibroblasts function. Volume of what elements is increased in this case?
  • 297. • A *Intercellular substance • B Vessels of microcircular stream • C Nerve elements • D Parenchymatous elements of the organ • E Lymphatic vessels •
  • 298. • The hyaluronidase increase occurs at a patient under the action of bacteria. How the intracellular substance permeability will change in this case?
  • 299. • A. *Increase of permeability • B. No effect • C. Reduce of permeability • D. Slowing of metabolism • E. Reduce the content of glycosaminoglycans
  • 300. • The local resorption of some bones hard tissues was noticed by the physician zt the roentgenogram of the patient 57 years old. With the increased activity of what cells can be associated these changes?
  • 301. • A * Osteoclasts • B Chondroblasts • C Osteocytes • D Osteoblasts • E Chondrocytes
  • 302. • The signs of regenerative process (callus) are present in the histological preparation of the bones. What tissue is forming the described structure?
  • 303. • A * Membrane reticulated bone • B Loose connective tissue • C Reticulartissue • D. Epithelial tissue • E Splenial bone
  • 304. • A tissue was represented in the histological slide. The cells of this tissue don’t have the processec. Each one of them conteins tens of nuclei and one surface with corrugated zone. Through this zone the secretion of hydrolytic enzymes is going. What tissue is presented in histological slide?
  • 305. • A * Bone tissue • B Cartilage tissue • C Epithelial tissue • D Nervous tissue • E Muscular tissue
  • 306. • The ossification resulted by the rise of the calcium level in the bones was revealed at the worker of the enterprise, which produces vanadium compounds. With activity of what cells can it be associated?
  • 307. • A * Osteoblast • B Osteocytes • C Osteoclasts • D Chondrocytes • E Fibroblasts •
  • 308. • The resorption of bone was revealed at the patient. With the increased activity of what cells can it be associated?
  • 309. • A * Osteoclast • B Osteoblasts and osteoclasts • C Osteocytes and osteoblasts • D Osteoblasts • E Osteocytes
  • 310. • The patients with a diagnosis of the clavicle fracture was admitted into a hospital. What cellular elements will take part in the regeneration of the bone tissue?
  • 311. • A * Osteoblasts • B Osteoclasts • C Osteocytes • D Chondrocytes • E Fibroblasts •
  • 312. • The excessive loss of bone tissue mass (it reflects the development of osteoporosis) is observed at the elderly people. Activation of what bone cells can cause the development of this disease?
  • 313. • A * Osteoclast • B Osteoblasts • C Macrophages • D Tissue basophils • E Osteocytes
  • 314. • The bones knitting at the fracture place is resulted by the osteoblasts activation. What is the function of these cells?
  • 315. A. A*Formation of the osteoid substance and transformation into osteocytes B. Formation of the connective tissue primary, from which will be bone formation C. Formation of the cartilage isogenic group D. Proliferation and calcification E. Formation of the bone and transformation into osteoclasts
  • 316. • The hydroxyapatite crystals deposition along the collagen fibers takes place during the intercellular substance of bone calcification. Presence of alkaline phosphatase in the intercellular substance in necessary for this process. What cell produces this enzyme?
  • 317. • A *Osteoblasts • B Osteocytes • C Osteoclasts • D Chondroblast • E Chondrocytes
  • 318. • The disorder of the motive functions resulted by the age changes of the hyaline cartilage was revealed at the patient 70 years old. What age changes caused limitation of the joints movement?
  • 319. • A * Deposition of calcium in the intercellular substance • B Increasing the number of isogenic groups • C Increasing the number of cartilage cells • D Thickening of the perichondrium • E Increasing the hydrophilicity of the ground substance
  • 320. • The isogenic cell groups were revealed at the histological slide of the cartilage. What cells are the primary in the formation of these groups?
  • 321. • A * Chondrocytes I type. • B Chondroblasts • C Prechondroblasts • D Chondrocytes II type • E Chondrocytes III type •
  • 322. • The tissue was represented in a histological slide. The cells of this tissue are situated singly or by isogroups and the fibrous structures of the intercellular substance are not visible. What tissue is represented in the slide?
  • 323. • A * Hyaline cartilage • B Smooth muscle tissue • C Epithelial tissue • D Fibrous cartilage • E Bone tissue •
  • 324. • The tissue is visible at the larynx tumor biopsy material. The cells of this tissue are situated singly or forming the isogenic cells group located in the same plane. The presence of elastic and collagen fibers is detected histologically. Which structures could develop this tumor?
  • 325. • A * Elastic cartilage • B Hyaline cartilage • C Fibrous cartilage • D Smooth muscle tissue • E Bone tissue
  • 326. • The two slides were proposed to the student. At the 1st one there is the elastic cartilage (stained by the orcein), at the 2nd one – the hyaline cartilage (stained by the hematoxylin-eosin). According to what features can we tell one from the other?
  • 327. • A * Presence of the elastic fibers • B Presence of the cells isogenic groups • C Presence of young cartilage zone • D Presence of the perichondrium • E Presence of amorphous substance
  • 328. • The elongation of the bones is stopped after the puberty. Decrease of what cell proliferation results stopping of the bones elongation?
  • 329. A. *Chondroblasts B. Osteocytes C. Osteoclasts D. Fibroblasts E. Neutrophils
  • 330. • During the study of histological preparations of the airways it was founded that the supporting structures of the large and medium bronchi wall is different according to their tissue composition. What tissue replaced the hyaline cartilage in the fibro- cartilaginous shell of secondary bronchi (compared to the large bronchi)?
  • 331. • A * Elastic cartilage • B Loose connective • C Hyaline cartilage • D Smooth muscle • E Fibrous cartilage
  • 332. • Synovial fluid normally doesn’t contains collagen. Analysis of synovial fluid of patients suffering from rheumatism, showed the presence of various types of collagen. Specify the type of erosive tissue if the synovial fluid is determined by collagen type II. •
  • 333. • A *Articular surface cartilage • B Synovial membrane • C Vascular endothelium • D Myocytes vessels • E Bone epiphysis •
  • 334. • The articular cartilage, as it is known, doesn’t have the perichondrium. What kind of growth is possible at this cartilage during the regeneration process?
  • 335. • A * Interstitial • B Appositional • C By application • D Appositional and interstitial • E It is not growing •
  • 336. • In course of indirect histogenesis of tubular bone tissue a plate is formed between epiphyseal and diaphyseal ossification centres that provides further lengthwise growth of bones. What structure is it?
  • 337. • A *Metaphyseal plate • B Osseous cuff • C Osseous plate • D Osteon • E Layer of interior general plates •
  • 338. • The disorder of cartilage tissue regeneration resulted by the injury of the undifferentiated cartilage cells was observed at the patient with severe upper extremity injury. What cells were injured?
  • 339. • A * Cells of the perichondrium inner layer • B Cells of the perichondrium outer layer • C Isogenic groups cells • D Cells of the young cartilage zone • E Cells originating from blood vessels
  • 340. • The cartilage part of the rib was damaged as result of trauma. At the cost of what layer of perichondrium the regeneration of the cartilage will take place?
  • 341. • A * Chondrogenic • B Fibrous • C Elastic • D Collagen • E Connective tissue
  • 342. • The destruction of the thin myofilaments is observed after the action of the hydrolytic enzymes. What structures were damaged?
  • 343. • A * Actin myofilaments • B Myosin myofilaments • C Tonofibrils • D Tropocollagen complexes • E Nucleoprotein complexes •
  • 344. • The tissue was represented at the histological slide. The structural unit of this tissue is the muscle fiber, which consist of the myosymplast and satellitocytes and is covered by a basement membrane. What tissue is characterized by this structure?
  • 345. • A * Skeletal striated muscle tissue • B Smooth muscle tissue • C Cardiac muscle tissue • D Loose connective tissue • E Reticular tissue •
  • 346. • Negative environmental factors have caused the dysfunction of myosatellite cells. What function of the whole muscle fiber is likely to be changed in this case?
  • 347. • A *Regeneration • B Contraction • C Trophism • D Contractile thermogenesis • E Relaxation •
  • 348. • The cesarean section was performed to a patient. The uterus wall was cut a significant extent and fetus released. By what mechanism the myometrium regeneration will be in the area of wound?
  • 349. • A *Formation of connective tissue scar • B Formation of new smooth muscle • C Formation of striated muscle fibers • D Proliferation of myosatellitocytes • E Hypertrophy of smooth muscle cells
  • 350. • A microspecimen of the submandibular salivary gland shows some basket-shaped cells concentrated around the acinus and excretory ducts. These cells surround bases of the serous cells and are called myoepitheliocytes. These cells relate to the following tissue:
  • 351. • A *Muscular tissue • B Epithelial tissue • C Neural tissue • D Special connective tissue • E Loose fibrous connective tissue
  • 352. • In course of a conditional experiment the development of mesenchyma cells was completely inhibited. Development of the following muscular tissue will be disturbed:
  • 353. • A *Smooth muscular tissue • B Neural muscular tissue • C Epidermal muscular tissue • D Cardiac muscular tissue • E Skeletal muscular tissue •
  • 354. • A big number of the intermediate microfilaments that contain desmin were revealed at the cytoplasm of the cell, which is spindle-shaped and has a rod-like nucleus. What tissue these cells belong to?
  • 355. • A * Muscle • B Nervous • C Epithelial • D Connective • E –
  • 356. • The destruction of the thick myofilaments is observed after mechanical injury of striated muscle fibers. Where is the localization of the pathological changes?
  • 357. • A * In the disk A • B In the disk I • C In the disk A half • D In the A and I disks • E In the disk I half •
  • 358. • Patient with injured muscles of the lower extremities was admitted to the traumatology department. Due to what cells is reparative regeneration of the muscle fibers and restoration of the muscle function possible?
  • 359. • A *Satellite-cells • B Myoblasts • C Myofibroblasts • D Fibroblasts • E Myoepithelial cells
  • 360. • It is known, that calcium ions, along with other factors, provide contraction of the muscle tissue. With what structures calcium interact during contraction?
  • 361. • A. *Protein troponin of thin fibrils • B. Protein myosin of thick fibrils • C. Protein actin of thin fibrils • D. Actomyosine complex of sarcolemma • E. Protein sequestrin
  • 362. • The skeletal muscle fibres are damaged due to injury. What structures can be the source of reparative regeneration of skeletal muscle fibres?
  • 363. • A. *Myosatellitocytes • B. Myofibrils • C. Miofilaments • D. Sarcolemma • E. Endomysium
  • 364. • Name the protein that is the main component of the thin myofilaments of the muscle cells. •
  • 365. A. *Actin B. Tubulin C. Dunein D. Desmin E. Keratin •
  • 366. • General increase of the skeletal muscle mass is takes place at the sportsman. Hypertrophy of what kind of the muscle fibers at the skeletal muscle will be dominating in this case?
  • 367. A. *White muscle fiber B. Red muscle fiber C. Intermediate muscle fiber D. Cardiac muscle fiber E. Smooth myocytes
  • 368. • A sensitive neural ganglion consists of roundish neurocytes with one extension that divides into axon and dendrite at some distance from the perikaryon. What are these cells called? •
  • 369. • A *Pseudounipolar • B Unipolar • C Bipolar • D Multipolar • E Apolar •
  • 370. • The toxic substances action violates the mechanism of the nerve impulses transmission at the experiment. What structure provides the implementation of this function?
  • 371. • A * Synapse • B Neurolemma • C Neurofibril • D Mitochondria • E Nissl’s substance •
  • 372. • The degeneration of nerve fibers accompanied by breakage of axial cylinders and myelin destruction may develop in case of traumatic injury of the upper limb. What neural structures will make the myelin renewal during the regeneration?
  • 373. • A * Neurolemmocytes (Schwann cells) • B Mesaxon • C Perineurium • D Endoneurium • E Astrocytes
  • 374. • The Nissl body lysis is observed in case of neuron functional exhaustion. It is resulted by decay of basophilic substance and brightening of the neuroplasma. With the changes of what organelle does this phenomenon is associated?
  • 375. • A. * Granular endoplasmic reticulum • B. Neurofibrils • C. Lysosomes • D. Golgi apparatus • E. Mitochondria
  • 376. • The emergency medical assistance was given to the patient, who had an accident. A tourniquet had been imposed forearm to stop the bleeding for a long time. This resulted in a loss of sensation and movement in the arm. Examination of the patient allowed to establish a violation of nerve conduction in a mixed nerve caused by its destruction without displacement of nerve fibers fragments. Proliferation of what cells will provide the axial cylinders regeneration in a mixed nerves?
  • 377. • A. * Schwann's cell • B. Fibrous astrocytes • C. Neurons • D. Ependymal cell • E. Protoplasmatic astrocytes •
  • 378. • To elderly patients turned to neurologist with complaints about the difficulty of the neck movements, numbness and loss of right hand sensation. Examination of the patient revealed a deformation of the spine in the lower cervical and upper thoracic part of the spinal cord caused by excessive deposition of calcium salts. What cells dysfunction resulted the loss of sensitivity in the hand?
  • 379. • A. * Pseudounipolar neurons • B. The neurons of the spinal cord anterior horns • C The neurons of the spinal cord posterior horns • D. The neurons of the spinal cord lateral horns • E. Pyramidal neurons •
  • 380. • The multipolar neurocytes are observed in the gray matter on the histological specimen of the cerebellum cross-section. According to what morphological features are these cells attributed to multipolar?
  • 381. • A * Amount of the processes • B Length of the processes • C Shape of the terminal axon extension • D Shape of the perikaryon • E Size of the cells •
  • 382. • The cells of neuroglia are forming from two sources: ectoderm and mesenchyme. What cells have the mesenchymal origin?
  • 383. • A * Microgliocytes • B Ependymal cell • C Fibrillar astrocytes • D Protoplasmatic astrocytes • E Oligodendrocyte
  • 384. • The spongioblasts were destroyed during the experiment. What violations of nerve tissue will occur during of it differentiation?
  • 385. • A *Disorder of neuroglia development • B Disorder of white matter of the spinal cord development • C Disorder of gray matter of the spinal cord development • D Disorder of the spinal ganglion development • E Disorder of the cerebrum development
  • 386. • The channel that is surrounded by a dense layer of cells we can see on the cross-section of the spinal cord. What are these cells and where they are located in the body?
  • 387. • A * Ependymal cells, all cerebrum ventricles • B Astrocytes, all cerebrum ventricles • C Oligodendrocytes, gray matter of the spinal cord • D Astrocytes, white matter of the spinal cord • E Neuron, myelin sheath •
  • 388. • The area of peripheral nerve longer than 10 cm was damaged after a mechanical injury. The motor activity of the upper extremity is impaired because of it. An allotransplantation of the nerve was proposed to the patient. What glial cells take will take a part in the regeneration of the damaged area.
  • 389. • A. * Neurolemmocytes • B. Fibrous astrocytes • C. Protoplasmatic astrocytes • D. Ependymal cells • E. Microgliocytes
  • 390. • The convulsions that associated with incomplete nerve fibers myelination can easily arise at the children of the first year. What glial cells most associated with this situation?
  • 391. • A. * Oligodendrocyte • B. Ependymal cells • C. Fibrous astrocytes • D. Microglia cells • E. Protoplasmatic astrocytes
  • 392. • The toxic substances action violates the mechanism of nerve impulses transmission in the experiment. What structure is provide the implementation of this function?
  • 393. • A * Synapse • B Neurolemma • C Neurofibrills • D Mitochondria • E Chromophilic substance
  • 394. • The concentrically oriented dark and bright rings, which are located around the axial cylinder, were revealed in the electron microscope pictures of the myelin fiber cross- section. What substances are forming the dark rings?
  • 395. • A *Lipids • B Protein • C Vitamins • D Carbohydrates • E Water •
  • 396. • The several axial cylinders with mesaxon were revealed in the electron microscope pictures of the fiber cross-section. What is the kind of fiber?
  • 397. • A * Nonmyelinated nerve fiber • B Reticular fiber • C Collagen fiber • D Elastic fiber • E Myelinated nerve fiber •
  • 398. • The fiber with one of its shell that is black and has a light notches located at an angle to the axial cylinder was founded in the nervous tissue slide colored by osmic acid. What is a shell?
  • 399. • A *Myelin • B Neurolemma • C Aksolemma • D Basement membrane • E Endonevrium
  • 401. • As a result of a trauma a patient has damaged anterior roots of spinal cord. What structures have been affected?
  • 402. • A *Axons of motoneurons and axons of neurons of lateral horns • B Central processes of sensitive neurons of spinal ganglions • C Peripheral processes of sensitive spinal ganglions • D Axons of neurons of lateral horns • E Dendrites of neurons of spinal ganglions
  • 403. • One of sections of central nervous system has layer wise arrangement of neurocytes. Among them there are cells of the following forms: stellate, fusiform, horizontal, pyramidal. What section of central nervous system is this structure typical for?
  • 404. • A Cortex of cerebrum • B Spinal cord • C Cerebellum • D Medulla oblongata • E Hypothalamus
  • 405. • As a result of an injury, the integrity of the anterior spinal cord root was broken. Specify the neurons and their processes that had been damaged:
  • 406. • A *Axons of motor neurons • B Motor neuron dendrites • C Axons of sensory neurons • D Dendrites of sensory neurons • E Dendrites of association neurons
  • 407. • Alcohol intoxication, as a rule, is accompanied by the coordination of movements’ disorder and imbalance caused by the damage of cerebellum structural elements. The function of what cells of cerebellum is affected first of all?
  • 408. A. *Purkinje’s cells B. Basket cells C. Betz cells D. Stellate cells E. Granule cells
  • 409. • Cerebellar cortex is revealed in a specimen impregnated with silver salts. It includes pyriform, basket, stellate and granule cells. What neurons constitute molecular layer?
  • 410. A. *Basket, small and large stellate cells B. Stellate and pyramidal cells C. Granule cells and large stellate cells D. Large stellate cells and spindle cells E. Pyriform cells
  • 411. • Cerebellar cortex is revealed in a specimen impregnated with silver salts. It includes pyriform, basket, stellate and granule cells. Which from mentioned above cells is efferent neuron of cerebellum?
  • 412. A. *Pyriform cells B. Stellate cells C. Granule cells D. Pyramidal cells E. Spindle cells
  • 413. • In a histological specimen an organ of nervous system is presented, which consists of grey and white substances. Grey substance is located on the periphery and consists of 6 layers: molecular, external granular, external pyramidal, internal granular, internal pyramidal and the layer of polymorphic cells.
  • 414. A. *Cerebral cortex B. Cerebellum C. Pons cerebelli D. Spinal ganglion E. Spinal cord
  • 415. • In a histological specimen an organ of nervous system is presented, which consists of grey and white substances. Grey substance is located on the center and consists of efferent neurons, projection neurons and interneurons. Name this organ.
  • 416. A. *Spinal cord B. Pons cerebelli C. Cerebral cortex D. Cerebellum E. Spinal ganglion
  • 417. • Parenchyma of the organ consists of the nervous tissue in which pseudounipolar neurons are revealed. Perikaryon of the neurons is covered with glial and connective tissue membranes and located in bunches. Name this organ.
  • 418. A. *Spinal ganglia (sensory ganglia) B. Vegetative ganglion C. Epiphysis D. Spinal cord E. Cerebellum
  • 419. • In a histological specimen an organ of nervous system is presented, which consists of grey and white substances. Grey substance is located on the center and forms butterfly. Neurons in the grey matter locate in bunches and form nucleuses. Which nucleus belongs to the central part of the vegetative nervous system?
  • 420. A. *Intermediate lateral nucleus B. Nucleus proprius of the dorsal horn C. Nucleus proprius of the ventral horn D. Nucleus thoracicus E. Intermediate medial nucleus •
  • 421. • Precentral gyrus section of the cerebral cortex is presented in the histological specimen. Indicate which layers mostly developed in this case.
  • 422. A. *Pyramidal external and internal and layer of polymorphic cells B. Molecular C. External and internal granular D. Molecular and layer of polymorphic cells E. Molecular, pyramidal external and internal
  • 423. • A part of the central nervous system has layer by layer allocation of neurocytes, among which there are cells of such forms: stellate, fusiform, horizontal, pyramidal. Which part of the CNS has this structure?
  • 424. A. *Cortex of large hemispheres B. Cerebellum C. Hypothalamus D. Medulla oblongata E. Spinal cord
  • 425. • 55-year- old patient has movements’ coordination and balance disorder as a result of permanent using of alcohol and developing intoxication. What nervous structures of the CNS have been disordered?
  • 426. A. *Purkinje cells of the cerebellum B. Basket cells of the cerebellum C. Stellate cells of the cerebellum D. Motor neurons of the spinal cord E. Olive of the medulla oblongata
  • 427. • 15-year-old patient enrolled in the clinic with diagnosis poliomyelitis. This disease is accompanied with disorder of movements. What nervous structures destruction can explain this disorder?
  • 428. A. *Motor neurons of the spinal cord B. Sensory neurons of the spinal ganglions C. Vegetative nucleuses of the spinal cord D. Substantia gelatinosa E. Neurons of the cerebellum
  • 429. • A patient with poliomyelitis (which characterized by spinal cord damage) has disorder of skeletal muscles function. What neurons destruction can explain this disorder?
  • 430. A. *Motor neurons B. Pseudounipolar C. Associative (interneurons) D. Pseudounipolar and associative (interneurons) E. Interneurons and motor •
  • 431. • In a histological specimen an organ of nervous system is presented, which consists of grey and white substances. Grey substance is located on the periphery. Neurons in the grey matter form three layers molecular, ganglionary (Purkinje), and granular. What organ has this structure?
  • 432. A. *Cerebellum B. Spinal cord C. Cerebral cortex D. Medulla oblongata E. Pons •
  • 433. • A specimen, dyed by the method of silver impregnation, is being investigated. Pyramidal cells of different size are seen in this specimen. Short processes come off their tips and lateral surfaces; one long process comes off the base of the cells. Name the specimen.
  • 434. A. *Cerebral cortex B. Spiral organ of the inner ear C. Retina of the eye D. Cortex of the cerebellum E. Spinal ganglion
  • 435. • A specimen, dyed by the method of silver impregnation, is being investigated. Pyriform cells with 2-3 climbing up prominent dendrites are seen in this specimen. Name the specimen.
  • 436. A. *Cortex of the cerebellum B. Spiral organ of the inner ear C. Retina of the eye D. Cerebral cortex E. Spinal ganglion •
  • 437. • During microscopic investigation of the CNS grey substance was revealed.. Neurons in it form three layers molecular, ganglionary (Purkinje), and granular. Name neurons which form second layer?
  • 438. A. *Pyriform cells B. Small stellate cells C. Granule cells D. Large stellate E. Basket cells •
  • 439. • In the microspecimen of the spinal cord nucleus neurons of which form motor ending in the skeletal muscles have to be analyzed. Indicate this nucleus
  • 440. A. *Nucleus proprius of the ventral horn B. Nucleus thoracicus C. Intermediolateral nucleus D. Nucleus proprius of the dorsal horn E. Nucleus proprius of grey substance
  • 441. • As a result of trauma anterior roots of the spinal cord were damaged in 47-year-old patient. Which neurons processes were damaged?
  • 442. A. *Axons of motor somatic and vegetative nucleuses neurons B. Axons of sensory pseudounipolar neurons C. Dendrites of sensory pseudounipolar neurons D. Dendrites of motor and axons of lateral horns nucleuses E. Dendrites and axons of sensory pseudounipolar neurons
  • 443. • A histological specimen presents a receptor zone of a sensoepithelial sense organ. Cells of this zone are placed upon the basal membrane and include the following types: external and internal receptor cells, external and internal phalangeal cell, stem cells, external limiting cells and external supporting cell. The described receptor zone belongs to the following sense organ: •
  • 444. • A *Acoustic organ • B Visual organ • C Gustatory organ • D Equilibrium organ • E Olfactory organ •
  • 445. • Vitamin A deficit results in the impairment of twilight vision. Name the cells that have the above- mentioned photoreceptor function:
  • 446. • A *Rod receptor cell • B Horizontal neurocytes • C Cone receptor cells • D Bipolar neurons • E Ganglion neurocytes •
  • 447. • An infectious disease caused contractive activity of muscles that contract and dilate eye pupil (paralytic state). What functional eye system was damaged?
  • 448. • A *Accomodative • B Dioptric • C Ancillary • D Photosensory • E Lacrimal apparatus •
  • 449. • A 14-year-old patient has twilight vision impairment. What vitamin deficit takes place in the organism?
  • 450. A. *A B. B1 C. В6 D. C E. В12
  • 451. • During examination an oculist has detected that patient has disorder of green color perception. Which retina cells absence may cause this sight disorder?
  • 452. • *Cone neurosensory cells • Rod neurosensory cells • Epithelial pigmented cells • Bipolar neuron • Ganglionic nerve cell
  • 453. • The increased intraocular tension is observed in the patient with glaucoma. Secretion of aqueous humor by the ciliary body is normal. Injury of what structure of the eyeball wall caused the disorder of flow-out from the anterior chamber?
  • 454. • A *Venous sinus • B Ciliary body • C Choroid • D Ciliary muscle • E Back epithelium of cornea
  • 455. • A histological specimen of the eyeball shows a biconvex structure connected to the ciliary body by the fibers of the Zinn's zonule and covered with a transparent capsule. Name this structure:
  • 456. • A *Crystalline lens • B Vitreous body • C Ciliary body • D Cornea • E Sclera •
  • 457. • In a histological specimen a structure of eyeball detected in which blood vessels are absent. What structure is characterized by this morphological sign?
  • 458. A. *Cornea B. Ciliary body C. Choroid D. Iris E. Retina •
  • 459. • In the electron microphotograph of the sense organ revealed cells peripheral part of which constitutes from two segments. The outer segment has membrane half discs and the inner one has ellipsoid. In what organ does this structure locate?
  • 460. A. *In the organ of vision B. In the organ of taste C. In the organ of smell D. In the vestibular organ E. In the auditory organ
  • 461. • In an electronic micrograph of a sense organ hair cells are seen, on their apical part there are short microvilli – stereocilii and a polar located kinocilium. For what sensory organ are such cells typical?
  • 462. A. *Vestibular organ B. Vision organ C. Olfactory organ D. Hearing organ E. Taste organ
  • 463. • Damages of vascular membrane are detected in the histological specimen of a fetus eyeball. What embryonic material was probably damaged during the development of the eye?
  • 464. A. *Mesenchyme B. Ectoderm C. Endoderm D. Outer layer of the eyeball E. Internal layer of the eyeball
  • 465. • In the electron microphotograph revealed cell of neural origin. Terminal part of the cell dendrite has cylindrical shape and consists from 1000 enclosed membrane discs. What cell is this?
  • 466. • *Rod cell • Ventral horn of the spinal cord • Sensory ganglia neuron • Cerebral cortex neuron • Cone cell •
  • 467. • As a result of trauma 30-year-old man has damaged mucosal membrane which covers upper part of upper conchae. To what consequences did this lead?
  • 468. • *Disorder of smell substances perception • Disorder of air moistening • Disorder of secretory activity of goblet cells • Disorder of air warming • Disorder of air moistening and warming
  • 469. • As a result of head trauma 32-year- old patient has damaged cristae ampullaris. Which stimuli perception have been disordered?
  • 470. • *Angular movement • Vibration • Gravitation • Linear movement • Vibration and gravitation •
  • 471. • A lot of people with age have clouding of the lens (phacoscotasmus or cataract) It became opaque that leading to the partial blindness. What optical and chemical properties of the lens fibers protein will be disordered?
  • 472.
  • 473. • *Crystalline • Vitrein • Dinein • Rhodopsin • Iodopsin
  • 474. • A patient has taken high doses of streptomycin and consequently became deaf. The function of what cells of the inner ear was damaged in this case?
  • 475. A. *Hair B. Phalangeal C. Pillar D. Deiters’ E. Connective tissue •
  • 476. • A boxer has disturbance in smell after a trauma of the nose. The damage of what cells may cause the loss of smell?
  • 477. A. *Neurosensory B. Supporting epithelial cells C. Basal epithelial cells D. Microvillous epithelial cells E. Brush cells
  • 478. • On the electron microphotograph neural origin cell is presented. This cell constitutes part of mucosa membrane epithelium. Distal part of peripheral process of the cell has knob-like expansion from which arise 10-15 cilia. What cell is it?
  • 479. A. *Olfactory cell B. Bipolar neuron of spinal ganglion C. Sensor epithelial cell of organ of taste D. Rode cell E. Cone cell
  • 480. • An infectious disease caused contractive activity of muscles that contract and dilate eye pupil (paralytic state). What functional eye system was damaged?
  • 481. • A *Accommodative • B Dioptric • C Ancillary • D Photosensory • E Lacrimal apparatus
  • 482. • Underdeveloped epithelium of cornea is observed in the histological specimen of a fetus’ eyeball. A part of what embryonal layer was probably affected in the process of embryogenesis?
  • 483. A. *Ectoderm B. Endoderm C. Mesoderm D. Outer layer of the eyeball E. Inner layer of the eyeball
  • 484. • Ciliary body was damaged in the patient. Function of what eye apparatus will suffer in this case?
  • 485. A. *Accommodative B. Protective C. Trophic D. Photosensitive E. Dioptric
  • 486. • Patient with eye trauma appealed to the doctor. During examination of corneal epithelium was revealed changes. What epithelium was damaged?
  • 487. A. *Stratified squamous non keratinized B. Simple pseudostratified C. Stratified squamous keratinized D. Stratified cuboidal E. Stratified cylindrical
  • 488. • In the histological specimen of the eyeball wall, structure which consists of three neurons chain is revealed. Bodies of these neurons form outer, inner nuclear and ganglion cell layers. Which eye component has this morphologic structure?
  • 489. A. *Retina B. Iris C. Sclera D. Choroid E. Ciliary body
  • 490. • Transplantation of the cornea was done to the patient. Which peculiarities of the cornea structure give expectation to engraftment of it?
  • 491. A. *Absence of blood and typical lymphatic vessels B. Presence of stratified anterior epithelium C. A huge innervations D. Presence of connective tissue E. Presence of simple squamous epithelium
  • 492. • Patient has appealed to the ophthalmologist with complaints of hurt in his eyes, which revealed itself after long presence of the patient in the field during dust storm. Doctor diagnosed superficial injuries of the external corneal epithelium. Which cells provide regeneration of the injured epithelium?
  • 493. A. *Basal cells B. Cells of stratum corneum C. Cells of stratum granulosum D. Cells of stratum spinosum E. Cells of stratum lucidum
  • 494. • Study of fingerprints (dactylography) is used by criminalists for personal identification as well as for diagnostics of genetic abnormalities, particularly Dawn's disease. What layer of skin determines individuality of fingerprints?
  • 495. • A *Dermopapillary • B Horny • C Reticular • D Clear (stratum lucidum epidermis) • E Basal •
  • 496. • A patient complains of dryness of head skin, itching, fragility and loss of hair. After examination he was diagnosed with seborrhea. Disturbed activity of which cells caused this condition?
  • 497. • A *Cells of sebaceous glands • B Cells of sudoriferous glands • C Epithelial cells • D Adipocytes • E Melanocytes •
  • 498. • In an electronic micrograph of skin epidermis among the cells of cuboidal form dendritic cells are detected. In their cytoplasm Golgi apparatus is well-developed; there are a lot of ribosomes and melanosomes. Name these cells.
  • 499. A. *Melanocytes B. Keratinocytes C. Cells of Langerhans D. Merkel’s cells E. Mast cells
  • 500. • Trauma of skin reticular layer was happened. At the expense of what cells differon activity regeneration of this layer will happen?
  • 501. A. *Fibroblastic B. Macrophagic C. Lymphoblastic D. Neuroblastic E. -
  • 502. • Under the radiation influence epidermal cells of the stratum basale were damaged. What function of the epidermis will be weakening or depressed first of all?
  • 503. A. *Regenerative B. Protective C. Absorptive D. Barrier E. Dielectric
  • 504. • With age human skin undergoes changes, which may declare themselves by reduction of skin elasticity. What structures of connective tissue provide skin elasticity most of all?
  • 505. A. *Collagen and elastic fibers B. Ground substance C. Epidermis cells D. Connective tissue cells E. Reticular fiber
  • 506. • In the course of experiment on a frog embryo the external embryonic layer – ectoderm – has been destroyed. Which of the following morphological structures has not been developed henceforth? •
  • 507. A. *Epidermis B. Somites C. Nephrotome D. Splanchnotome E. Sclerotome
  • 508. • One of the surgery rules is performing sections along the so- called Langers’ lines (lines of skin tension). Which from mentioned below tissues form papillary layer (the strongest layer in the derma)?
  • 509. A. *Dense irregular connective tissue B. Reticular connective tissue C. Loose connective tissue D. Epithelial tissue E. Dense regular connective tissue
  • 510. • A child has abraded skin of the palm when falling down. What epithelium was damaged?
  • 511. A. *Stratified keratinized B. Stratified non-keratinized C. Simple low columnar D. Transitional E. Simple squamous
  • 512. • A patient complains of dryness of head skin, itching, fragility and loss of hair. After examination he was diagnosed with seborrhea. Disturbed activity of which cells caused this condition?
  • 513. • A *Cells of sebaceous glands • B Cells of sudoriferous glands • C Epithelial cells • D Adipocytes • E Melanocytes
  • 514. • Cells with processes and dark brown granules in the cytoplasm were revealed in the skin epidermis of the biopsy material. What cell are these?
  • 515. A. *Melanocytes B. Intraepidermal macrophages C. Keratinocytes D. Merkel cells E. Lymphocytes
  • 516. • Some layers are absent on a limited area of epidermis after a trauma. Only germinative layer is preserved. Name the cells, which will become the main source of its regeneration.
  • 517. A. *Layer of basal cells B. Layer of spinosum cells C. Layer of granulosum cells D. Layer of spinous and granular cells of undisturbed area E. Cells of lucidum layer of undisturbed area
  • 518. • A 12-year-old patient has white spots without a pigment on skin. Spots have appeared after the age of 10, constantly increase in size. The absence of what cells of skin led to such formations appearance?
  • 519. A. *Melanocytes B. Adipocytes C. Fibrocytes D. Plasma cells E. Mast cells
  • 520. • During the third week of embryogenesis the central part of epiblast cells (ectoderm) sags and neurulation process begins. In which direction will the remaining ectodermal cells differentiate?
  • 521. A. *Skin B. Somites C. Gut D. Chord E. Yolk sac
  • 522. • There is histological specimen of the skin epidermis bioptic sample taken from the healthy adult. Cells division can be seen in the stratum basale. What process does provide these cells?
  • 523. A. *Physiologic regeneration B. Differentiation C. Adaptation D. Reparation E. Apoptosis
  • 524. • In forensics method of dactyloscopy is largely used. This method is based on papillary layer of derma which determines strictly individual print on the skin surface. Which tissue forms this layer of the derma?
  • 525. • *Loose irregular connective tissue • Reticular connective tissue • Dense irregular connective tissue • Adipose tissue • Dense regular connective tissue
  • 526. • Stratified structure organ which covered with stratified keratinized squamous epithelium is represented in the histological specimen. There is loose connective tissue which forms convexity in the form of papilla located under epithelial basal membrane. Dense irregular connective tissue located under it and form reticular layer. What organ has these morphological peculiarities?
  • 527. A. *Skin B. Tongue C. Esophagus D. Tonsil E. Cervix of the uterus
  • 528. • There are next layers in the skin histological specimen: stratum basale, spinosum, granulosum, lucidum and corneum. Which part of the human body does this epithelium belong?
  • 529. A. *Palm skin B. Face skin C. Hairy part of the head skin D. Shoulder skin E. Thigh skin
  • 530. • Skin malignant tumor was revealed of 30-year-old patient. What epidermis cells take place in the immune response?
  • 531. A. *T-lymphocytes B. Keratinocytes C. Keratinocytes and Merkel cells D. Merkel cells E. Stratum spinosum cells •
  • 532. • Process of dorsal mesoderm segmentation and somites formation was disordered in the embryo. In what part of the skin disorders of development are possible?
  • 533. A. *Derma B. Hair C. Sebaceous glands D. Epidermis E. Sweat glands
  • 534. • Trauma of derma reticular layer was happened. At the expense of what cell differon activity regeneration of this layer will happen?